Sunteți pe pagina 1din 53

Part 2_Case Digests

3.1_Insurance Law

D. Insurable Interest - Mhh

33) SPOUSES NILO CHA and STELLA UY CHA, and UNITED INSURANCE
CO., INC., petitioners, vs. COURT OF APPEALS and CKS DEVELOPMENT
CORPORATION, respondents (GR No. 124520, 1997)
Facts: Sps Cha leased CKS Dev Corps premises, the contract includes:
The LESSEE shall not insure against fire the chattels, merchandise,
textiles, goods and effects placed at any stall or store or space in
the leased premises without first obtaining the written consent and
approval of the LESSOR. If the LESSEE obtain(s) the insurance
thereof without the consent of the LESSOR then the policy is
deemed assigned and transferred to the LESSOR for its own
benefit.
Sps Cha insured against loss by fire their merchandise inside the leased premises
for P500,000 with United Insurance, without the written consent of CKS. Fire broke
out. Upon learning of the insurance, CKS wrote United asking the proceeds.
United refused to pay CKS.
RTC ordered United to pay CKS, and Sps Cha also to pay CKS for exemplary
damages etc.
CA affirmed, but deleted the order of payment by Sps Cha.
Issue: WON the contract stipulation was valid?
Held: No. CKS has no insurable interest in the goods inside the leased premises,
pursuant to:
Section 17, Insurance Code. The measure of an insurable interest in
property is the extent to which the insured might be damnified by
loss of injury thereof.
A non-life insurance policy such as the fire insurance policy taken by Spouses
over their merchandise is primarily a contract of indemnity. Insurable interest in
the property insured must exist at the time the insurance takes effect and at the
time the loss occurs. The basis of such requirement of insurable interest in
property insured is based on sound public policy: to prevent a person from taking
out an insurance policy on property upon which he has no insurable interest and
collecting the proceeds of said policy in case of loss of the property. In such a
case, the contract of insurance is a mere wager which is void under Section 25 of
the Insurance Code.

1
The automatic assignment of the policy to CKS under the provision of the lease
contract previously quoted is void for being contrary to law and/or public policy.
The proceeds of the fire insurance policy thus rightfully belong to the spouses.
34) GREAT PACIFIC LIFE ASSURANCE CORP., petitioner, vs. COURT OF
APPEALS AND MEDARDA V. LEUTERIO, respondents (GR No. 113899, 1999)
Facts: Grepalife agreed to insure the lives of eligible housing loan mortgagors of
DBP, under a contract of group life insurance. Dr. Wilfredo Leuterio, a physician
and a housing debtor of DBP applied for membership in the group life insurance
plan. In an application form, Dr. Leuterio answered he was in good health.
Grepalife insured Dr. Leuterio to the extent of his DBP mortgage indebtedness of
P86,200. Later, Dr. Leuterio died due to massive cerebral hemorrhage. DBP
claimed from Grepalife, but was denied payment. Grepalife insisted that Dr.
Leuterio did not disclose he had been suffering from hypertension, which caused
his death. Allegedly, such non-disclosure constituted concealment that justified
the denial of the claim.
Mrs. Leuterio filed a complaint with RTC for specific performance. Dr. Mejia, the
one who issued the death certificate, testified that Dr. Leuterio was not autopsied
so other causes were not ruled out, even though the deceased complained of
headaches presumably due to high blood. RTC ruled in favor of the widow. CA
affirmed the RTC decision.
Issue: Can the widow, not the real party in interest, file the suit against
Grepalife?
Held: Yes. Since a policy of insurance upon life or health may pass by transfer,
will or succession to any person, whether he has an insurable interest or not.
Such person may recover whatever the insured might have recovered. The
insured private respondent did not cede to the mortgagee all his rights or
interests in the insurance, the policy stating that: "In the event of the debtor's
death before his indebtedness with the DBP shall have been fully paid, an amount
to pay the outstanding indebtedness shall first be paid to the creditor and the
balance of sum assured, if there is any, shall then be paid to the beneficiaries
designated by the debtor. The insured may be regarded as the real party in
interest, although he has assigned the policy for the purpose of collection, or has
assigned as collateral security any judgment he may obtain.
Side Notes: The rationale of a group insurance policy of mortgagors, otherwise
known as the "mortgage redemption insurance," is a device for the protection of
both the mortgagee and the mortgagor. On the part of the mortgagee, it has to
enter into such form of contract so that in the event of the unexpected demise of
the mortgagor during the subsistence of the mortgage contract, the proceeds

Part 2_Case Digests


3.1_Insurance Law

from such insurance will be applied to the payment of the mortgage debt, thereby
relieving the heirs of the mortgagor from paying the obligation. Ample protection
is given to the mortgagor under such a concept so that in the event of death; the
mortgage obligation will be extinguished by the application of the insurance
proceeds to the mortgage indebtedness. Consequently, where the mortgagor
pays the insurance premium under the group insurance policy, making the loss
payable to the mortgagee, the insurance is on the mortgagor's interest, and the
mortgagor continues to be a party to the contract. In this type of policy insurance,
the mortgagee is simply an appointee of the insurance fund, such loss-payable
clause does not make the mortgagee a party to the contract.
As to concealment, the fraudulent intent on the part of the insured must be
established to entitle the insurer to rescind the contract. Misrepresentation as a
defense of the insurer to avoid liability is an affirmative defense and the duty to
establish such defense by satisfactory and convincing evidence rests upon the
insurer. This Grepalife failed.
35) Harvardian Colleges v. Country Bankers Insurance Corp . (1 CARA 2) from the net
Facts: Harvardian is a family corporation, the stockholders of which are Ildefonso
Yap, Virginia King Yap and their children. Prior to Aug. 9, 1979, an agent of
Country Bankers proposed to Harvardian to insure its school building. Although at
first reluctant, Harvardian agreed. Country Banks sent an inspector to inspect the
school building and agreed to insure the same for P500,000 for which Harvardian
paid an annual premium of P2,500. On Aug. 9, 1979, Country Bankers issued to
Harvardian a fire insurance policy. On March 12, 1980, during the effectivity of
said insurance policy, the insured property was totally burned rendering it a total
loss. A claim was made by plaintiff upon defendant but defendant denied it
contending that plaintiff had no insurable interest over the building constructed
on the piece of land in the name of the late Ildefonso Yap as owner. It was
contended that both the lot and the building were owned by Ildefonso Yap and
NOT by the Harvardian Colleges.
Issue: Whether or not Harvardian colleges has a right to the proceeds.
Held: Harvardian has a right to the proceeds. Regardless of the nature of the title
of the insured or even if he did not have title to the property insured, the contract
of fire insurance should still be upheld if his interest in or his relation to the
property is such that he will be benefited in its continued existence or suffer a
direct pecuniary loss from its destruction or injury. The test in determining
insurable interest in property is whether one will derive pecuniary benefit or
advantage from its preservation, or will suffer pecuniary loss or damage from its
destruction, termination or injury by the happening of the event insured against.

2
Here Harvardian was not only in possession of the building but was in fact using
the same for several years with the knowledge and consent of Ildefonso Yap. It is
reasonably fair to assume that had the building not been burned, Harvardian
would have been allowed the continued use of the same as the site of its
operation as an educational institution. Harvardian therefore would have been
directly benefited by the preservation of the property, and certainly suffered a
pecuniary loss by its being burned.
36) ANG KA YU v. PHOENIX ASSURANCE CO . (GR No. 27881, 1961) from the
net
Facts: Ang Ka Yu and Kee Boc are joint owners of a business establishment,
known as Ang Ka Yu Dyeing and Bleaching Factory, in Malabon Rizal. A deed of
sale was allegedly made in favor of Francisco Monroy covering the building of said
business establishment. Ang Ka Yu and Kee Boc obtained a fire insurance policy
covering the property from Phoenix Assurance Co. Franciso Monroy likewise
insured the same property with British Traders Insurance Co., Commonwealth
Insurance Co., and Great American insurance Co. The insured property was
consumed by fire, so Ang Ka Yu sought to claim the proceeds of the policy.
Phoenix Assurance Co. denied liability on the ground that petitioner was not the
owner but a mere possessor and as such, had no insurable interest over the
property.
Issue: Whether or not a mere possessor has insurable interest over the property?
Held: Yes. A person having a mere right or possession of property may insure it to
its full value and in his own name, even when he is not responsible for its
safekeeping. The reason is that even if a person is NOT interested in the safety
and preservation of material in his possession because they belong to third
parties, said person still has insurable interest, because he stands either to
benefit from their continued existence or to be prejudiced by their destruction.
Petitioners insurable interest is not limited to their liability to the owners of
insured materials. They are interested in the safety and preservation of said
materials. Destruction of the textiles would mean a pecuniary loss to petitioners
because they were deprived of the compensation they will receive or are entitled
to for dyeing the same.

E. Concealment and Representation


37) THE INSULAR LIFE ASSURANCE CO., LTD., petitioner, vs. SERAFIN D.
FELICIANO and ANGEL, FLORENDA, EUGENIO, HERMINIO and LETICIA, all
surnamed FELICIANO, represented by their guardian ad litem SERAFIN
D. FELICIANO, respondents (GR No. 47593, 1941)

Part 2_Case Digests


3.1_Insurance Law
Facts: Evaristo Feliciano was granted 2 insurance policies to the aggregate
amount of P25,000. Later, he died. The Insular Life refused to pay on the ground
that the policies were fraudulently obtained, the insured having given false
answers and statements in the application as well as in the medical report.
The lower court ruled in favor of the plaintiffs. At the time Feliciano filed his
application and at the time he was subjected to physical examination by the
medical examiner of the herein petitioner, he was already suffering from
tuberculosis. This fact appears in the negative both in the application and in the
medical report. The lower court also found that Feliciano was made to sign the
application and the examiner's report in blank, and that afterwards the blank
spaces therein were filled in by the agent and the medical examiner, who made it
appear therein that Feliciano was a fit subject for insurance. The lower court also
held that neither the insured nor any member of his family concealed the real
state of health of the insured. That as a matter of fact the insured, as well as the
members of his family, told the agent and the medical examiner that the
applicant had been sick and coughing for some time and that he had also gone
three times to the Santol Sanatorium. On appeal, this finding of facts of the lower
court was sustained by the Court of Appeals.
Issue: Who should bear the loss arising from misrepresentation?
Held: That as between the two of them, the one who employed and gave
character to the third person as its agent should be the one to bear the loss. In
the present case, the agent knew all the time the true state of health of the
insured. The insurer's medical examiner approved the application knowing full
well that the applicant was sick. The situation is one in which one of two innocent
parties must bear a loss for his reliance upon a third person. In this case, it was
the insurer who gave the agent authority to deal with the applicant. It was the
one who selected the agent, thus implying that the insured could put his trust on
him. It was the one who drafted and accepted the policy and consummated the
contract.
If an agent of the insurer, after obtaining from an applicant for insurance a correct
and truthful answer to interrogatories contained in the application for insurance,
without knowledge of the applicant fills in false answers, either fraudulently or
otherwise, the insurer cannot assert the falsity of such answers as a defense to
liability on the policy, and this is true generally without regard to the subject
matter of the answers or the nature of the agent's duties or limitations on his
authority, at least if not brought to the attention of the applicant.
The fact that the insured did not read the application which he signed, is not
indicative of bad faith. It has been held that it is not negligence for the insured to

3
sign an application without first reading it if the insurer by its conduct in
appointing the agent influenced the insured to place trust and confidence in the
agent. In the instant case, it has been proved that the insured could not read
English, the language in which the application was written, and that after the
contract was signed, it was kept by his mother. As a consequence, the insured
had no opportunity to read or correct any misstatement therein.
38) THE INSULAR LIFE ASSURANCE CO., LTD., petitioner, vs. SERAFIN D.
FELICIANO ET AL., respondents (GR No. 47593, 1943)
- continuation of previous case
Facts: Evaristo Feliciano, who died, was suffering with advanced pulmonary
tuberculosis when he signed his application for insurance with the petitioner.
Doctor Trepp, who had taken X-ray pictures of his lungs, informed the respondent
Dr. Serafin D. Feliciano, brother of Evaristo, that the latter "was already in a very
serious and practically hopeless condition." Nevertheless the question contained
in the application "Have you ever suffered from any ailment or disease of the
lungs, pleurisy, pneumonia or asthma?" appears to have been answered, "No."
And above the signature of the applicant, following the answers to the various
questions propounded to him, is the following printed statement:
I declare on behalf of myself and of any person who shall have or
claim any interest in any policy issued hereunder, that each of the
above answers is full, complete and true, and that to the best of my
knowledge and belief I am a proper subject for life insurance.
The false answer above referred to, as well as the others, was written by the
Company's soliciting agent Romulo M. David, in collusion with the medical
examiner Dr. Gregorio Valdez, for the purpose of securing the Company's
approval of the application so that the policy to be issued thereon might be
credited to said agent in connection with the inter-provincial contest which the
Company was then holding among its soliciting agents to boost the sales of its
policies. Agent David bribed Medical Examiner Valdez.
Issue: What is the effect of misrepresentation?
Held: The insured was a co-participant, and co-responsible with Agent David and
Medical Examiner Valdez, in the fraudulent procurement of the policies in
question and that by reason thereof said policies are void ab initio.
Although the agent and the medical examiner knew that statement to be false, no
valid contract of insurance was entered into because there was no real meeting of
the minds of the parties. The policies were issued on the basis of the statement
subscribed by the applicant to the effect that he was and had been in good
health, when as a matter of fact he was then suffering from advanced pulmonary
tuberculosis.

Part 2_Case Digests


3.1_Insurance Law

When Evaristo Feliciano, the applicant for insurance, signed the application in
blank and authorized the soliciting agent and/or the medical examiner of the
Company to write the answers for him, he made them his own agents for that
purpose, and he was responsible for their acts in that connection. If they falsified
the answers for him, he could not evade the responsibility for the falsification. He
was not supposed to sign the application in blank. He knew that the answers to
the questions therein contained would be "the basis of the policy," and for that
very reason he was required with his signature to vouch for the truth thereof.
39) SUNLIFE ASSURANCE COMPANY OF CANADA, petitioner, vs. The Hon.
COURT OF APPEALS and Spouses ROLANDO and BERNARDA BACANI,
respondents (GR No. 105135, 1995)
Facts: Robert Bacani insured himself with Sunlife. His policy valued P100,000
with double indemnity in case of accidental death, the beneficiary of which is his
mother, Bernarda. Robert died in a plane crash. Bernarda claimed from Sunlife
but was denied payment. It said that the insured did not disclose material facts
relevant to the issuance of the policy. Robert disclosed that within the past 5
years, he has consulted with a certain Dr. Reinaldo D. Raymundo of the Chinese
General Hospital for cough and flu complications. He answered the other
questions (as to being submitted to ECG, x-rays, blood tests or other tests and
WON he was admitted to any hospital) in the negative. Sunlife discovered that
Robert was examined and confined at the Lung Center of the Philippines, for renal
failure. He was subjected to urinalysis etc.
RTC held that the facts concealed were made in good faith and under belief that
they need not be disclosed. It further held that the health history of the insured
was immaterial since the insurance policy was non-medical. CA affirmed the RTC
decision.
Issue: May Sunlife rescind the contract of insurance due to the concealment?
Held: Yes. Section 27 of the Insurance Code allows the injured party to rescind a
contract of insurance where there is concealment. Further, as in this case,
rescission was exercised within the two-year contestability period as recognized in
Section 48 of The Insurance Code.
Section 26 of The Insurance Code is explicit in requiring a party to a contract of
insurance to communicate to the other, in good faith, all facts within his
knowledge which are material to the contract and as to which he makes no
warranty, and which the other has no means of ascertaining. Said Section
provides: "A neglect to communicate that which a party knows and ought to
communicate, is called concealment."

4
Materiality is to be determined not by the event, but solely by the probable and
reasonable influence of the facts upon the party to whom communication is due,
in forming his estimate of the disadvantages of the proposed contract or in
making his inquiries (Sec. 31). The terms of the contract are clear. The insured is
specifically required to disclose to the insurer matters relating to his health. The
information which the insured failed to disclose were material and relevant to the
approval and issuance of the insurance policy. The matters concealed would have
definitely affected petitioner's action on his application, either by approving it
with the corresponding adjustment for a higher premium or rejecting the same.
Moreover, a disclosure may have warranted a medical examination of the insured
by petitioner in order for it to reasonably assess the risk involved in accepting the
application. Materiality of the information withheld does not depend on the state
of mind of the insured. Neither does it depend on the actual or physical events
which ensue. Thus, "good faith" is no defense in concealment. The insured's
failure to disclose the fact that he was hospitalized for two weeks prior to filing his
application for insurance, raises grave doubts about his bonafides. It appears that
such concealment was deliberate on his part.
The waiver of a medical examination in a non-medical insurance contract renders
even more material the information required of the applicant concerning previous
condition of health and diseases suffered, for such information necessarily
constitutes an important factor which the insurer takes into consideration in
deciding whether to issue the policy or not.
Anent the finding that the facts concealed had no bearing to the cause of death of
the insured the insured need not die of the disease he had failed to disclose to the
insurer. It is sufficient that his non-disclosure misled the insurer in forming his
estimates of the risks of the proposed insurance policy or in making inquiries.
40) THELMA VDA. DE CANILANG, petitioner, vs. HON. COURT OF APPEALS
and GREAT PACIFIC LIFE INSURANCE CORPORATION , respondents (GR No.
92492, 1993)
Facts: Jaime Canilang consulted with Dr. Claudio and was diagnosed as suffering
from sinus tachycardia." Mr. Canilang consulted the same doctor again and was
found to have "acute bronchitis." Thereafter, Jaime applied for a "non-medical"
insurance policy with respondent Grepalife naming his wife, petitioner Thelma, as
his beneficiary. He was issued ordinary life insurance policy with the face value of
P19,700. A year later, Jaime died of congestive heart failure, anemia, and chronic
anemia. Widow Thelma, filed a claim with Grepalife but was denied payment,
saying the insured had concealed material information from it, when he declared:

Part 2_Case Digests


3.1_Insurance Law

(1) I have not been confined in any hospital, sanitarium or


infirmary, nor received any medical or surgical advice/attention
within the last five (5) years
(2) I have never been treated nor consulted a physician for a heart
condition, high blood pressure, cancer, diabetes, lung, kidney,
stomach disorder, or any other physical impairment.
(3) I am, to the best of my knowledge, in good health.
He also did not state under the Exceptions part the 2 consultation with
Dr. Claudio.
Insurance Commissioner ordered Grepalife to pay the widow, saying the failure to
convey certain info was not intentional. CA reversed decision, dismissing
Thelmas complaint.
Issue: Should the concealment be intentional?
Held: No. The unspoken theory of the Insurance Commissioner appears to have
been that by deleting the phrase "intentional or unintentional," the Insurance
Code of 1978 (prior to its amendment by B.P. Blg. 874) intended to limit the kinds
of concealment which generate a right to rescind on the part of the injured party
to "intentional concealments." The deletion of the phrase could not have had the
effect of imposing an affirmative requirement that a concealment must be
intentional if it is to entitle the injured party to rescind a contract of insurance.
From 1914 up to 1974, under Act 2427:
Sec. 26. A concealment, whether intentional or unintentional,
entitles the injured party to rescind a contract of insurance."
From 1974 up to 1985, under the Insurance Code of 1978:
Sec. 27. A concealment entitles the injured party to rescind a
contract of insurance.
From 1985 onwards, under the Insurance Code of 1978 as amended by BP
874:
Sec. 27. A concealment whether intentional or unintentional
entitles the injured party to rescind a contract of insurance.
But in any rate, Jaime cannot be said to have not disclosed the information
inadvertently. He could not have been unaware that his heart beat would
rise to high and alarming levels at times because of his sinus tachycardia.
Sec. 26. A neglect to communicate that which a party knows and
ought to communicate, is called a concealment.

5
Sec. 28. Each party to a contract of insurance must communicate
to the other, in good faith, all factors within his knowledge which
are material to the contract and as to which he makes no warranty,
and which the other has not the means of ascertaining.
Under the foregoing provisions, the information concealed must be information
which the concealing party knew and "ought to have communicated," that is to
say, information which was "material to the contract." The test of materiality is
contained in Section 31 of the Insurance Code of 1978 which reads:
Sec. 31. Materiality is to be determined not by the event, but solely
by the probable and reasonable influence of the facts upon the
party to whom the communication is due, in forming his estimate of
the disadvantages of the proposed contract, or in making his
inquiries.
41) PHILAMCARE HEALTH SYSTEMS, INC. , petitioner, vs. COURT OF
APPEALS and JULITA TRINOS, respondents (GR No. 125678, 2002)
- see case no. 19
Synopsis: Ernani Trinos answered in the negative Have you or any of your
family members ever consulted or been treated for high blood pressure, heart
trouble, diabetes, cancer, liver disease, asthma or peptic ulcer? He was issued a
Health Care Agreement for a health coverage with petitioner. During the period of
his coverage, he suffered a heart attack and was confined in the hospital.
Respondent tried to claim the benefits under the health care agreement, but
petitioner denied her claim saying there was a concealment regarding Ernani's
medical history. Doctors at the MMC allegedly discovered at the time of Ernani's
confinement that he was hypertensive, diabetic and asthmatic, contrary to his
answer in the application form. Respondent paid the hospitalization expenses of
P76,000 herself. Ernani was again confined to the hospital where he died
eventually.
Respondent then filed with the RTC an action for damages against petitioner and
its president, Dr. Benito Reverente. The court ruled in favor of Julita and awarded
damages. On appeal, the Court of Appeals affirmed the decision of the trial court
but deleted all awards for damages and absolved petitioner Reverente. Hence,
petitioner brought the instant petition for review, raising the primary argument
that a health care agreement is not an insurance contract.
In affirming the decision of the Court of Appeals, the Supreme Court ruled that an
insurance contract exists when the following elements concur: (1) the insured has
an insurable interest; (2) the insured is subject to a risk of loss by the happening
of the designated peril; (3) the insurer assumes the risk; (4) such assumption of

Part 2_Case Digests


3.1_Insurance Law

risk is part of a general scheme to distribute actual losses among a large group of
persons bearing a similar risk; and (5) in consideration of the insurer's promise,
the insured pays a premium.
The health care agreement was in the nature of a non-life insurance, which
is primarily a contract of indemnity.
The answer assailed by petitioner was in response to the question relating to the
medical history of the applicant. This largely depends on opinion rather than fact,
especially coming from respondent's husband who was not a medical doctor.
Where matters of opinion or judgment are called for, answers made in good faith
and without intent to deceive will not avoid a policy even though they are untrue.
Thus, "Although false, a representation of the expectation, intention, belief,
opinion, or judgment of the insured will not avoid the policy if there is no actual
fraud in inducing the acceptance of the risk, or its acceptance at a lower rate of
premium, and this is likewise the rule although the statement is material to the
risk, if the statement is obviously of the foregoing character, since in such case
the insurer is not justified in relying upon such statement, but is obligated to
make further inquiry. There is a clear distinction between such a case and one in
which the insured is fraudulently and intentionally states to be true, as a matter
of expectation or belief, that which he then knows, to be actually untrue, or the
impossibility of which is shown by the facts within his knowledge, since in such
case the intent to deceive the insurer is obvious and amounts to actual fraud."
The fraudulent intent on the part of the insured must be established to warrant
rescission of the insurance contract. Concealment as a defense for the health care
provider or insurer to avoid liability is an affirmative defense and the duty to
establish such defense by satisfactory and convincing evidence rests upon the
provider or insurer.
Under Section 27 of the Insurance Code, "a concealment entitles the injured party
to rescind a contract of insurance." The right to rescind should be exercised
previous to the commencement of an action on the contract. In this case, no
rescission was made. Besides, the cancellation of health care agreements as in
insurance policies require the concurrence of the following conditions:
1. Prior notice of cancellation to insured;
2. Notice must be based on the occurrence after effective date of the policy
of one or more of the grounds mentioned;
3. Must be in writing, mailed or delivered to the insured at the address shown
in the policy;
4. Must state the grounds relied upon provided in Section 64 of the Insurance
Code and upon request of insured, to furnish facts on which cancellation is
based.
None of the above pre-conditions was fulfilled in this case.

Part 2_Case Digests


3.1_Insurance Law

F. Persons entitled to recover under the policy


42) Bonifacio Bros., Inc. v. Mora, 20 SCRA 261
Facts: Enrique Mora mortgaged his Odlsmobile sedan car to HS Reyes Inc. with
the condition that Mora would insure the car with HS Reyes as beneficiary. The car
was then insured with State Insurance Company and the policy delivered to Mora.
During the effectivity of the insurance contract, the car figured in an accident.
The company then assigned the accident to an insurance appraiser for
investigation and appraisal of the damage. Mora without the knowledge and
consent of HS Reyes, authorized Bonifacio Bros to fix the car, using materials
supplied by the Ayala Auto Parts Company. For the cost of Labor and materials,
Mora was billed P2,102.73. The bill was sent to the insurers appraiser. The
insurance company drew a check in the amount of the insurance proceeds and
entrusted the check to its appraiser for delivery to the proper party. The car was
delivered to Mora without the consent of HS Reyes, and without payment to
Bonifacio Bros and Ayala. Upon the theory that the insurance proceeds should be
directly paid to them, Bonifacio and Ayala filed a complaint against Mora and the
insurer with the municipal court for the collection of P2,102.73. The insurance
company filed its answer with a counterclaim for interpleader, requiring Bonifacio
and HS Reyes to interplead in order to determine who has a better right to the
proceeds.
Issue: Whether or not there is privity of contract between Bonficacio and Ayala
on one hand and State Insurance on the other.
Held: NONE.
It is fundamental that contracts take effect only between the parties thereto,
except in some specific instance provided by law where the contract contains
some stipulation in favor of a third person. Such stipulation is known as a
stipulation pour autrui; or a provision in favor of a third person not a party to the
contract.
Under this doctrine, a third person is ed to avail himself of a benefit granted to
him by the terms of the contract, provided that the contracting parties have
clearly and deliberately conferred a favor upon such person. Consequently, a
third person NOT a party to the contract has NO action against the aprties
thereto, and cannot generally demand the enforcement of the same.
The question of whether a third person has an enforceable interest in a contract
must be settled by determining whether the contracting parties intended to

7
tender him such an interest by deliberately inserting terms in their agreement
with the avowed purpose of conferring favor upon such third person. IN this
connection, this court has laid down the rule that the fairest test to determine
whether the interest of a 3rd person in a contract is a stipulation pour autrui or
merely an incidental interest, is to rely upon the intention of the parties as
disclosed by their contract.
In the instant case the insurance contract does not contain any words or clauses
to disclose an intent to give any benefit to any repairmen or material men in case
of repair of the car in question. The parties to the insurance contract omitted such
stipulation, which is a circumstance that supports the said conclusion. On the
other hand, the "loss payable" clause of the insurance policy stipulates that "Loss,
if any, is payable to H.S. Reyes, Inc." indicating that it was only the H.S. Reyes,
Inc. which they intended to benefit.
A policy of insurance is a distinct and independent contract between the insured
and insurer, and third persons have no right either in a court of equity, or in a
court of law, to the proceeds of it, unless there be some contract of trust,
expressed or implied, by the insured and third person. In this case, no contract of
trust, express or implied. In this case, no contract of trust, expressed or implied
exists. We, therefore, agree with the trial court that no cause of action exists in
favor of the appellants in so far as the proceeds of insurance are concerned. The
appellant's claim, if at all, is merely equitable in nature and must be made
effective through Enrique Mora who entered into a contract with the Bonifacio
Bros Inc. This conclusion is deducible not only from the principle governing the
operation and effect of insurance contracts in general, but is clearly covered by
the express provisions of section 50 of the Insurance Act (now Sec. 53).
The policy in question has been so framed that "Loss, if any, is payable to H. S.
Reyes, Inc." which unmistakably shows the intention of the parties.
43) First Integrated Bonding & Insurance Co. v. Hernando, 199 SCRA 796
Facts: Silverio Blanco was the owner of a passenger jeepney which he insured
against liabilities for death andinjuries to third persons with First Integrated
Bonding and Insurance Company, Inc. for P30,000. The said jeepneydriven by
Blanco himself bumped a five-year old child, Deogracias Advincula, causing the
latter's death. The boysparents filed a complaint for damages against Blanco and
First Insurance, which was granted by the lower court. FirstInsurance filed a
petition for certiorari contending that the victims parents have no cause of action
against it becausethey are not parties to the insurance contract and that they

Part 2_Case Digests


3.1_Insurance Law

may only proceed against the driver based on theprovisions of the New Civil
Code.
Issue: W/N an injured party for whom the contract of insurance is intended can
sue directly the insurer
Held: YES. Where the insurance contract provides for indemnity against liability
to a third party, such third party can directly sue the insurer. The liability of the
insurer to such third person is based on contract while the liability of the insured
to the third party is based on tort. It cannot evade its liability as insurer by hiding
under the cloak of the insured. Its liability is primary and not dependent on the
recovery of judgment from the insured.

G. Incontestable Clause - Abellana

45) Tan v. Court of Appeals, 174 SCRA 403

Facts: Shafer obtained a private car policy over his Ford Laser car from Makati
Insurance Compani, Inc. for Third Party Liability (TPL). The car hit a Volkswagen
owned by Legazpi, causing also physical injuries to its passenger Poblete. Shafer
filed a third party complaint against Makati Insurance but the latter moved to
dismiss the same. The RTC dismissed the case.

Facts: Tan Lee Siong, father of herein Tans, applied for life insurance in the
amount of P80,000 with respondent company. Said application was approved and
Policy No. 1082467 was issued effective November 6, 1973, with Tans the
beneficiaries thereof. Tan Lee Siong died of hepatoma. Tans then filed with
respondent company their claim for the proceeds of the life insurance policy.
However, in a letter respondent company denied Tans' claim and rescinded the
policy by reason of the alleged misrepresentation and concealment of material
facts made by the deceased Tan Lee Siong in his application for insurance. The
premiums paid on the policy were thereupon refunded.

Issue: WON the insured can file a third party complaint in the criminal case filed
against him.

The Tans filed a complaint before the Insurance Commission. It rendered


judgment against Tan. CA affirmed.

Held: Yes. A third party complaint is a device allowed by the rules of procedure by
which the defendant can bring into the original suit a party against whom he will
have a claim for indemnity or remuneration as a result of a liability established
against him in the original suit. The injured party did not reserve his right to file a
separate civil action, thus, the same was deemed instituted with the criminal
action. Shafer may thus raise all defenses available to him insofar as the criminal
and civil aspects of the case are concerned. The injured, for whom the contract of
insurance is intended, can sue directly the insurer. In the event that the injured
fails or refuses to include the insurer as part defendant in his claim for indemnity
against the insured, the latter is not prevented by law to avail of the procedural
rules intended to avoid multiplicity of suits. Not even a no action clause under
the policy which requires that a final judgment be fist obtained against the
insured and that only thereafter can the person insured recover on the policy can
prevail over the Rules of Court provisions aimed at avoiding multiplicity of suits.

Issue: WON the insurer company no longer had the right to rescind the contract
of insurance as rescission must allegedly be done during the lifetime of the
insured within two years and prior to the commencement of action.

44) Sherman Shafer v. Hon. Judge, RTC of Olongapo, 167 SCRA 386

Held: No, this stand is incorrect. The insurer has two years from the date of
issuance of the insurance contract or of its last reinstatement within which to
contest the policy, whether or not, the insured still lives within such period. After
two years, the defenses of concealment or misrepresentation, no matter how
patent or well founded, no longer lie. Congress felt this was a sufficient answer to
the various tactics employed by insurance companies to avoid liability. The Tans'
interpretation would give rise to the incongruous situation where the beneficiaries
of an insured who dies right after taking out and paying for a life insurance policy,
would be allowed to collect on the policy even if the insured fraudulently
concealed material facts.
The so-called "incontestability clause" precludes the insurer from raising the
defenses of false representations or concealment of material facts insofar as

Part 2_Case Digests


3.1_Insurance Law

health and previous diseases are concerned if the insurance has been in force for
at least two years during the insured's lifetime. The phrase "during the lifetime"
found in Section 48 simply means that the policy is no longer considered in force
after the insured has died. The key phrase in the second paragraph of Section 48
is "for a period of two years.
The pertinent section in the Insurance Code provides:
Section 48.Whenever a right to rescind a contract of insurance is given to the
insurer by any provision of this chapter, such right must be exercised
previous to the commencement of an action on the contract.
After a policy of life insurance made payable on the death of the insured
shall have been in force during the lifetime of the insured for a period of two
years from the date of its issue or of its last reinstatement, the insurer
cannot prove that the policy is void ab initio or is rescindible by reason of the
fraudulent concealment or misrepresentation of the insured or his agent.
The policy was issued on November 6, 1973 and the insured died on April 26,
1975. The policy was thus in force for a period of only one year and five
months. Considering that the insured died before the two-year period had
lapsed, respondent company is not, therefore, barred from proving that the policy
is void ab initio by reason of the insured's fraudulent concealment or
misrepresentation. Moreover, respondent company rescinded the contract of
insurance and refunded the premiums paid on September 11, 1975, previous to
the commencement of this action on November 27, 1975.
The evidence for respondent company shows that on September 19, 1972, the
deceased was examined by Dr. Victoriano Lim and was found to be diabetic and
hypertensive; that by January, 1973, the deceased was complaining of
progressive weight loss and abdominal pain and was diagnosed to be suffering
from hepatoma. Another physician, Dr. Wenceslao Vitug, testified that the
deceased came to see him on December 14, 1973 for consultation and claimed to
have been diabetic for five years. Because of the concealment made by the
deceased of his consultations and treatments for hypertension, diabetes and liver
disorders, respondent company was thus misled into accepting the risk and
approving his application as medically standard and dispensing with further
medical investigation and examination. For as long as no adverse medical history
is revealed in the application form, and applicant for insurance is presumed to be
healthy and physically fit and no further medical investigation or examination is
conducted by respondent company.

H. Liability under Open Policy - Mhh


46)
DEVELOPMENT
INSURANCE
CORPORATION, petitioner, vs. INTERMEDIATE APPELLATE COURT,
and
PHILIPPINE
UNION
REALTY
DEVELOPMENT
CORPORATION, respondents
(GR No. L- 71360, 1986)
Facts: Philippine Union (PURDC) insured its 7-storey building from fire with
Development Insurance Corp. (DIC). Fire broke out. DIC did not pay due to the
issue of coverage of the elevators in said insurance. PURDC sued to recover for
the damages. DIC failed to answer on time and was declared in default by the
RTC. A judgment of default granting full recovery of PURDC based on ex-parte
evidence. Upon learning, DIC moved to lift the order of default, invoking
excusable neglect. Denied.
CA affirmed decision in toto.
Issues:
1. Was the order of default proper?
2. Is DIC liable to pay the proceeds? If so, how much?
Held:
1. Yes. It is indisputable that summons was served on DIC, through its senior
vice-president, on June 19, 1980. 10 days after the expiration of the
original 15-day period to answer, its counsel filed an ex parte motion for

Part 2_Case Digests


3.1_Insurance Law

an extension of 5 days within which to file its answer. On July 18, 1980, the
last day of the requested extension which at the time had not yet been
granted the same counsel filed a second motion for another 5-day
extension, fourteen days after the expiry of the original period to file its
answer. The trial court nevertheless gave it five days from July 14, 1980, or
until July 19, 1980, within which to file its answer. But it did not. It did so
only or 21 days after the July 5, deadline. As a consequence, the trial
court, on motion of the private respondent filed on July 28, 1980, declared
the petitioner in default. This was done almost one month later, on August
25, 1980. Even so, the petitioner made no move at all for two months
thereafter. It was only on October 27, 1980, more than one month after the
judgment of default was rendered by the trial court on September 26,
1980, that it filed a motion to lift the order of default and vacate the
judgment by default. The petitioner has slumbered on its right and
awakened too late. Although judgments by default are generally looked
upon with disfavor, but here, summons was served on the vice-president
of the petitioner who however refused to accept it.

2. Yes. The petitioner's claim that the insurance covered only the building
and not the elevators is absurd. This Court has little patience with puerile
arguments that affront common sense, let alone basic legal principles with
which even law students are familiar. The circumstance that the building
insured is seven stories high and so had to be provided with elevators a
legal requirement known to the petitioner as an insurance company
makes its contention all the more ridiculous.
The petitioner even expressed admission in its answers affirming that the fire
"damaged or destroyed a portion of the 7th floor of the insured building and more
particularly a Hitachi elevator control panel."
There is no reason to disturb the factual findings of the lower court, as affirmed
by the Intermediate Appellate Court, that the heat and moisture caused by the
fire damaged although they did not actually burn the elevators. Neither is this
Court justified in reversing their determination, also factual, of the value of the
loss sustained by the private respondent in the amount of P508,867.
Although the building was insured at P2.5 million (to be presumed as
actual value on the day the fire broke out), but since this is an open
policy, only P508,867 is the amount of indemnity to PURDC.
The insurance policy was an open policy and is subject to the express condition
that:
Open Policy.

10
This is an open policy as defined in Section 57 of the Insurance
Act. In the event of loss, whether total or partial, it is understood
that the amount of the loss shall be subject to appraisal and the
liability of the company, if established, shall be limited to the
actual loss, subject to the applicable terms, conditions, warranties
and clauses of this Policy, and in no case shall exceed the amount
of the policy.
As defined in the aforestated provision, which is now Section 60 of the Insurance
Code, "an open policy is one in which the value of the thing insured is not
agreed upon but is left to be ascertained in case of loss." This means that
the actual loss, as determined, will represent the total indemnity due
the insured from the insurer except only that the total indemnity shall
not exceed the face value of the policy.
The actual loss has been ascertained in this case and, this Court will respect such
factual determination in the absence of proof that it was arrived at arbitrarily.
There is no such showing. Hence, applying the open policy clause as expressly
agreed upon by the parties in their contract, the Philippine Union is entitled to the
payment of indemnity under the said contract in the total amount of P508,867.

I. Prescription of Action - Abellana


47) SUN INSURANCE OFFICE, LTD., petitioner v COURT OF APPEALS and
EMILIO TAN, respondents 8/20/1983 Tan wrote to Sun >> Sun refused on
2/29/1984 (received by Tan on April 2, 1984)>>Tan wrote again on
4/3/1984>>9/15/1985 Tans cousel wrote to Sun>>Sun answered on
10/11/1985>>Tan FILED before the RTC on 11/20/1985 (then Suns motion to
dismiss was denied on 11/3/1987 and on 1/14/1988); From the first denial by Sun
on 2/29/1984 which was received by Tan on 4/2/1984 to the filing of the action on
11/20/1985, 20 months had elapsed.
Facts: On August 15, 1983, herein private respondent Emilio Tan took from herein
Sun Insurance a P300,000 property insurance policy to cover his interest in the
electrical supply store of his brother housed in a building in Iloilo City. Four (4)
days after the issuance of the policy, the building was burned including the
insured store. On August 20, 1983, Tan filed his claim for fire loss with Sun
Insurance, but on February 29, 1984, Sun Insurance wrote Tan denying the latter's
claim. On April 3, 1984, Tan wrote Sun Insurance, seeking reconsideration of the
denial of his claim. On September 3, 1985, Tan's counsel wrote the Insurer
inquiring about the status of his April 3, 1984 request for reconsideration. Sun
Insurance answered the letter on October 11, 1985, advising Tan's counsel that
the Insurer's denial of Tan's claim remained unchanged, enclosing copies of Sun

Part 2_Case Digests


3.1_Insurance Law

Insurances' letters of February 29, 1984 and May 17, 1985 (response to petition
for reconsideration).
Tan filed before the RTC against Sun Insurance and the RTC denied the latters
motion to dismiss on the ground that the action had already prescribed. Sun
sought to nullify RTCs order before the CA but the latter denied it.
Hence, the instant petition.
Issues:
1. WON, the filing of a motion for reconsideration (between the insurer and the
insured) interrupts the 12 months prescriptive period to contest the denial of
the insurance claim.
2. WON, the rejection of the claim shall be deemed final only if it contains words
to the effect that the denial is final.
Held/Ratio:
1. No, while it is a cardinal principle of insurance law that a policy or contract of
insurance is to be construed liberally in favor of the insured and strictly
against the insurer company, yet, contracts of insurance, like other contracts,
are to be construed according to the sense and meaning of the terms which
the parties themselves have used. If such terms are clear and unambiguous,
they must be taken and understood in their plain, ordinary and popular sense.
Condition 27 of the Insurance Policy, which is the subject of the conflicting
contentions of the parties, reads:
27. Action or suit clause If a claim be made and rejected and an action or
suit be not commenced either in the Insurance Commission or in any court of
competent jurisdiction within twelve (12) months from receipt of notice of
such rejection, or in case of arbitration taking place as provided herein, within
twelve (12) months after due notice of the award made by the arbitrator or
arbitrators or umpire, then the claim shall for all purposes be deemed to have
been abandoned and shall not thereafter be recoverable hereunder.
As the terms are very clear and free from any doubt or ambiguity whatsoever, it
must be taken and understood in its plain, ordinary and popular sense.
Tan, in his letter addressed to the Sun Insurance insurance company
dated April 3, 1984 admitted that he received a copy of the letter of
rejection on April 2, 1984. Thus, the 12-month prescriptive period
started to run from the said date of April 2, 1984, for such is the plain
meaning and intention of Section 27 of the insurance policy.

11
The condition contained in an insurance policy that claims must be presented
within one year after rejection is not merely a procedural requirement but an
important matter essential to a prompt settlement of claims against insurance
companies as it demands that insurance suits be brought by the insured while the
evidence as to the origin and cause of destruction have not yet disappeared.
The contention of the respondents that the one-year prescriptive period does not
start to run until the petition for reconsideration had been resolved by the insurer:
runs counter to the declared purpose for requiting that an action or suit be
filed in the Insurance Commission or in a court of competent jurisdiction
from the denial of the claim
would contradict and defeat the very principle which this Court had laid
down
can easily be used by insured persons as a scheme or device to waste
time until any evidence which may be considered against them is
destroyed
It is apparent that Section 27 of the insurance policy was stipulated pursuant to
Section 63 of the Insurance Code, which states that:
Sec. 63. A condition, stipulation or agreement in any policy of insurance,
limiting the time for commencing an action thereunder to a period of less
than one year from the time when the cause of action accrues, is void.
The crucial issue in this case is: When does the cause of action accrue?
- when ones right to file a claim either in the Insurance Commission or in a court
of competent jurisdiction commences from the time of the denial of his claim by
the Insurer, either expressly or impliedly. Rejection in the first instance.
While in the Eagle Star case (96 Phil. 701), this Court uses the phrase "final
rejection", the same cannot be taken to mean the rejection of a petition for
reconsideration as insisted by respondents. Such was clearly not the meaning
contemplated by this Court. The Insurance policy in said case provides that the
insured should file his claim, first, with the carrier and then with the insurer. The
"final rejection" being referred to in said case is the rejection by the insurance
company.
48) Jacqueline Vda. De Gabriel v. Court of Appeals, 264 SCRA 137
Facts: Marcelino Gabriel, the insured, was employed by Emerald Construction &
Development Corporation (ECDC) at its construction project in Iraq. He was
covered by a personal accident insurance in the amount of P100,000 under a
group policy procured from Fortune Insurance by ECDC for its overseas workers.
The insured risk was for "bodily injury caused by violent accidental external and

Part 2_Case Digests


3.1_Insurance Law

visible means which injury (would) solely and independently of any other cause
result in death or disability.
On 22 May 1982, within the life of the policy, Gabriel died in Iraq. A year later, or
on 12 July 1983, ECDC reported Gabriel's death to Fortune Insurance by
telephone. Among the documents thereafter submitted to Fortune Insurance were
a copy of the death certificate issued by the Ministry of Health of the Republic of
Iraq which stated
REASON OF DEATH: UNDER EXAMINATION NOW NOT YET KNOWN

and an autopsy report of the National Bureau of Investigation ("NBI") to the


effect that "(d)ue to advanced state of postmortem decomposition, cause of
death (could) not be determined."
Following a series of communications between Jacqueline De Gabriel and Fortune
Insurance, the latter, on 22 September 1983, ultimately denied the claim of ECDC
on the ground of prescription.
Petitioner filed a complaint with RTC. She averred that her husband died of
electrocution while in the performance of his work and prayed for the recovery of
P100,000 for insurance indemnification and of various other sums by way of
actual, moral, and exemplary damages, plus attorney's fees and costs of suit.
Fortune Insurance filed its answer, which was not verified, admitting the
genuineness and due execution of the insurance policy; it alleged, however, that
since both the death certificate issued by the Iraqi Ministry of Health and the
autopsy report of the NBI failed to disclose the cause of Gabriel's death, it denied
liability under the policy. In addition, Fortune Insurance raised the defense of
"prescription," invoking Section 384 of the Insurance Code. Later, Fortune
Insurance filed an amended answer, still unverified, reiterating its original
defenses but, this time, additionally putting up a counterclaim and a crossclaim.
The RTC ruled in favor of Jacqueline; in arriving at its conclusion, the trial court
held that Fortune Insurance was deemed to have waived the defense, i.e., that
the cause of Gabriel's death was not covered by the policy, when the latter failed
to impugn by evidence petitioner's averment on the matter. With regard to the
defense of prescription, the court considered the complaint to have been timely
filed or within one (1) year from Fortune Insurance's denial of the claim.
The CA reversed. The appellate court held that petitioner had failed to
substantiate her allegation that her husband's death was caused by a risk insured
against. The appellate court observed that the only evidence presented by
petitioner, in her attempt to show the circumstances that led to the death of the
insured, were her own affidavit and a letter allegedly written by a co-worker of the
deceased in Iraq which, unfortunately for her, were held to be both

12
hearsay.
Issue: WON, Jacqueline De Gabriels cause of action prescribed.
Held: Yes. It had prescribed Fortune Insurance correctly invoked Section 384 of
the Insurance Code:
Sec. 384. Any person having any claim upon the policy issued
pursuant to this chapter shall, without any unnecessary delay,
present to the insurance company concerned a written notice of
claim setting forth the nature, extent and duration of the injuries
sustained as certified by a duly licensed physician. Notice of claim
must be filed within six months from date of the accident,
otherwise, the claim shall be deemed waived. Action or suit
for recovery of damage due to loss or injury must be brought,
in proper cases, with the Commissioner or the Courts within
one year from denial of the claim, otherwise, the claimant's
right of action shall prescribe.
The notice of death was given to Fortune Insurance, concededly, more than a
year after the death of petitioner's husband. Fortune Insurance, in invoking
prescription, was not referring to the one-year period from the denial of the claim
within which to file an action against an insurer but obviously to the written notice
of claim that had to be submitted within six months from the time of the accident.
The insurance policy expressly provided that to be compensable, the injury or
death should be caused by "violent accidental external and visible means." All
that petitioner could submit were a letter sent to her by her husband's co-worker,
stating that Gabriel died when he tried to haul water out of a tank while its
submerged motor was still functioning, and petitioner's sinumpaang salaysay
which merely confirmed the receipt and stated contents of the letter. The said
affidavit however suffers from procedural infirmity as it was not even testified to
or identified by the affiant (plaintiff-appellant) herself. This self-serving affidavit
therefore is a mere hearsay under the rules. In like manner, the letter allegedly
written by the deceased's co-worker which was never identified to in court by the
supposed author, suffers from the same defect as the affidavit of the plaintiffappellant.
Not one of the other documents submitted, to wit, the POEA decision, dated 06
June 1984, the death certificate issued by the Ministry of Health of Iraq and the
NBI autopsy report, could give any probative value to petitioner's claim. The POEA
decision did not make any categorical holding on the specific cause of Gabriel's
death. Neither did the death certificate issued by the health authorities in Iraq nor
the NBI autopsy report provide any clue on the cause of death. All that appeared
to be clear was the fact of Gabriel's demise on 22 May 1982 in Iraq.

Part 2_Case Digests


3.1_Insurance Law
Evidence, in fine, is utterly wanting to establish that the insured suffered from an
accidental death, the risk covered by the policy. In an accident insurance, the
insured's beneficiary has the burden of proof in demonstrating that the
cause of death is due to the covered peril. Once that fact is established,
the burden then shifts to the insurer to show any excepted peril that
may have been stipulated by the parties. An "accident insurance" is not thus
to be likened to an ordinary life insurance where the insured's death, regardless
of the cause thereof, would normally be compensable. The latter is akin in
property insurance to an "all risk" coverage where the insured, on the aspect of
burden of proof, has merely to show the condition of the property insured when
the policy attaches and the fact of loss or damage during the period of the policy
and where, thereafter, the burden would be on the insurer to show any "excluded
peril." When, however, the insured risk is specified, like in the case before us, it
lies with the claimant of the insurance proceeds to initially prove that the loss is
caused by the covered peril.

13
Held: No, there was none.

On procedural aspect:
The appeal filed by MICO was beyond the reglementary period as between the
two dates, the court chooses to believe June 13, 1982, not only because the
numbers "6-13-82" appear on both annexes but also because it is the date
authenticated by the administrative division of the Insurance Commission. Annex
"B" is at worst self-serving; at best, it might only indicate that it was received on
June 18, 1982, by the legal department of MICO, after it had been received earlier
by some other of its personnel on June 13, 1982. Whatever the reason for the
delay in transmitting it to the legal department need not detain us here.

49) Malayan Insurance Co. v. Cruz Arnaldo, 154 SCRA 672


Facts: On June 7, 1981, the Malayan Insurance (MICO) issued to, Coronacion
Pinca, Fire Insurance Policy on her property for the amount of P14,000 effective
July 22, 1981, until July 22, 1982.

Under Section 416 of the Insurance Code, the period for appeal is thirty days from
notice of the decision of the Insurance Commission. The MICO filed its motion for
reconsideration on April 25, 1981, or fifteen days such notice, and the
reglementary period began to run again after June 13, 1981, date of its receipt of
notice of the denial of the said motion for reconsideration. As the herein petition
was filed on July 2, 1981, or nineteen days later, there is no question that it is
tardy by four days.

On October 15, 1981, MICO allegedly cancelled the policy for non-payment, of the
premium and sent the corresponding notice to Pinca. Payment of the premium
for Pinca was received by Domingo Adora, agent of MICO. On January 15, 1982,
Adora remitted this payment to MICO, together with other payments. On January
18, 1982, Pinca's property was completely burned. On February 5, 1982,
Pinca's payment was returned by MICO to Adora on the ground that her policy
had been cancelled earlier. But Adora refused to accept it.

Counted from June 13, the fifteen-day period prescribed under Rule 45, assuming
it is applicable, would end on June 28, 1982, or also four days from July 2, when
the petition was filed.

J. Premium Payments - Abellana

In due time, Pinca made the requisite demands for payment, which MICO
rejected. She then went to the Insurance Commission. It is because she was
ultimately sustained by the Arnaldo (Insurance Commissioner) that MICO has
come to us for relief.

Issue: WON, there was a valid cancellation of the policy.

If it was filed under B.P. 129, then, considering that the motion for
reconsideration was filed on the fifteenth day after MICO received notice of the
decision, only one more day would have remained for it to appeal, to wit, June 14,
1982. That would make the petition eighteen days late by July 2.

Indeed, even if the applicable law were still R.A. 5434, governing appeals from
administrative bodies, the petition would still be tardy. The law provides for a
fixed period of ten days from notice of the denial of a seasonable motion for
reconsideration within which to appeal from the decision. Accordingly, that ten-

Part 2_Case Digests


3.1_Insurance Law

day period, counted from June 13, 1982, would have ended on June 23, 1982,
making the petition filed on July 2, 1982, nine days late.

Whichever law is applicable, therefore, the petition can and should be dismissed
for late filing.

14
December 24, 1981 but only up to July 22, 1981, according to the original terms.
In others words, the policy would have run for only eight months although the
premium paid was for one whole year.

MICO's acknowledgment of Adora as its agent defeats its contention that he was
not authorized to receive the premium payment on its behalf. It is clearly
provided in Section 306 of the Insurance Code that:

On substantial aspect:

SEC. 306. xxx xxx xxx

MICO's arguments that there was no payment of premium and that the policy had
been cancelled before the occurence of the loss are not acceptable. Its contention
that the claim was allowed without proof of loss is also untenable.

Any insurance company which delivers to an insurance agent or insurance


broker a policy or contract of insurance shall be deemed to have authorized
such agent or broker to receive on its behalf payment of any premium
which is due on such policy or contract of insurance at the time of its
issuance or delivery or which becomes due thereon.

The petitioner relies heavily on Section 77 of the Insurance Code providing that:
SEC. 77. An insurer is entitled to payment of the premium as soon as the thing
is exposed to the peril insured against. Notwithstanding any agreement to the
contrary, no policy or contract of insurance issued by an insurance company is
valid and binding unless and until the premium thereof has been paid, except
in the case of a life or an industrial life policy whenever the grace period
provision applies.

The above provision is not applicable because payment of the premium was in
fact eventually made in this case. The payment was made on December 24,
1981, and the fire occured on January 18, 1982.

And it is a well-known principle under the law of agency that:


Payment to an agent having authority to receive or collect payment is
equivalent to payment to the principal himself; such payment is complete
when the money delivered is into the agent's hands and is a discharge of
the indebtedness owing to the principal.

Payment was in fact made, rendering the policy operative as of June 22, 1981,
and removing it from the provisions of Article 77, Thereafter, the policy could be
cancelled on any of the supervening grounds enumerated in Article 64 (except
"nonpayment of premium") provided the cancellation was made in accordance
therewith and with Article 65.

One wonders: suppose the payment had been made and accepted in, say, August
1981, would the commencement date of the policy have been changed to the
date of the payment, or would the payment have retroacted to July 22, 1981? If
MICO accepted the payment in December 1981 and the insured property had not
been burned, would that policy not have expired just the same on July 22, 1982,
pursuant to its original terms, and not on December 24, 1982?

Section 64 reads as follows:

It would seem from MICO's own theory that the policy would have become
effective only upon payment, if accepted and so would have been valid only from

(a) non-payment of premium;

No policy of insurance other than life shall be cancelled by the insurer except
upon prior notice thereof to the insured, and no notice of cancellation shall be
effective unless it is based on the occurrence, after the effective date of the
policy, of one or more of the following:

Part 2_Case Digests


3.1_Insurance Law

(b) conviction of a crime arising out of acts increasing the hazard insured against;
(c) discovery of fraud or material misrepresentation;
(d) discovery of willful, or reckless acts or commissions increasing the hazard
insured against;
(e) physical changes in the property insured which result in the property
becoming uninsurable;or
(f) a determination by the Commissioner that the continuation of the policy would
violate or would place the insurer in violation of this Code.

As for the method of cancellation, Section 65 provides as follows:


All notices of cancellation mentioned in the preceding section shall be in writing,
mailed or delivered to the named insured at the address shown in the policy, and
shall state (a) which of the grounds set forth in section sixty-four is relied upon
and (b) that, upon written request of the named insured, the insurer will furnish
the facts on which the cancellation is based.
A valid cancellation must, therefore, require concurrence of the following
conditions:
(1) There must be prior notice of cancellation to the insured;
(2) The notice must be based on the occurrence, after the effective date of the
policy, of one or more of the grounds mentioned;
(3) The notice must be (a) in writing, (b) mailed, or delivered to the named
insured, (c) at the address shown in the policy;
(4) It must state (a) which of the grounds mentioned in Section 64 is relied upon
and (b) that upon written request of the insured, the insurer will furnish the facts
on which the cancellation is based.

MICO's claims it cancelled the policy in question on October 15, 1981, for nonpayment of premium. To support this assertion, it presented one of its employees,
who testified that "the original of the endorsement and credit memo"
presumably meaning the alleged cancellation "were sent the assured by mail
through our mailing section" However, there is no proof that the notice, assuming

15
it complied with the other requisites mentioned above, was actually mailed to and
received by Pinca.

On the other hand, there is the flat denial of Pinca, who says she never received
the claimed cancellation and who, of course, did not have to prove such denial.
50) Makati Tuscany Condominium Corp. v. Court of Appeals, 215 SCRA
462
Facts: Sometime in early 1982, American Home Assurance Co. (AHAC),
represented by American International Underwriters (Phils.), Inc., issued in favor
of Makati Tuscany Condominium Corporation (TUSCANY) Insurance Policy No. AHCPP-9210452 on the latter's building and premises, for a period beginning 1
March 1982 and ending 1 March 1983, with a total premium of P466,103.05. The
premium was paid on installments on 12 March 1982, 20 May 1982, 21 June 1982
and 16 November 1982, all of which were accepted by private respondent.
On 10 February 1983, AHAC issued to Tuscany Insurance Policy No. AH-CPP9210596, which replaced and renewed the previous policy, for a term covering 1
March 1983 to 1 March 1984. The premium in the amount of P466,103.05 was
again paid on installments on 13 April 1983, 13 July 1983, 3 August 1983, 9
September 1983, and 21 November 1983. All payments were likewise accepted
by AHAC.
On 20 January 1984, the policy was again renewed and AHAC issued to Tuscany
Insurance Policy No. AH-CPP-9210651 for the period 1 March 1984 to 1 March
1985. On this renewed policy, Tuscany made two installment payments, both
accepted by AHAC, the first on 6 February 1984 for P52,000.00 and the second,
on 6 June 1984 for P100,000.00. Thereafter, Tuscany refused to pay the balance
of the premium.
Consequently, AHAC filed an action to recover the unpaid balance of P314,103.05
for Insurance Policy No. AH-CPP-9210651.
In its answer with counterclaim, Tuscany admitted the issuance of Insurance
Policy No. AH-CPP-9210651. It explained that it discontinued the payment of
premiums because the policy did not contain a credit clause in its favor and the
receipts for the installment payments covering the policy for 1984-85, as well as
the two (2) previous policies, stated the following reservations:
2. Acceptance of this payment shall not waive any of the company rights to
deny liability on any claim under the policy arising before such payments or
after the expiration of the credit clause of the policy; and

Part 2_Case Digests


3.1_Insurance Law

3. Subject to no loss prior to premium payment. If there be any loss such is


not covered.
Tuscany further claimed that the policy was never binding and valid, and no risk
attached to the policy. It then pleaded a counterclaim for P152,000.00 for the
premiums already paid for 1984-85, and in its answer with amended
counterclaim, sought the refund of P924,206.10 representing the premium
payments for 1982-85.
On 8 October 1987, the trial court dismissed the complaint and the counterclaim
upon the following findings:
While it is true that the receipts issued to the defendant contained the
aforementioned reservations, it is equally true that payment of the premiums of
the three aforementioned policies (being sought to be refunded) were made
during the lifetime or term of said policies, hence, it could not be said, inspite of
the reservations, that no risk attached under the policies. Consequently,
defendant's counterclaim for refund is not justified.
As regards the unpaid premiums on Insurance Policy No. AH-CPP-9210651, in view
of the reservation in the receipts ordinarily issued by the AHAC on premium
payments the only plausible conclusion is that AHAC has no right to demand their
payment after the lapse of the term of said policy on March 1, 1985. Therefore,
the Tuscany was justified in refusing to pay the same.
The CA modified the decision and held Tuscany to pay the balance of the
premiums plus legal interest and affirmed the denial of the counterclaim. The CA
rationcinated that the obligation to pay premiums when due is ordinarily as
indivisible obligation to pay the entire premium. Here, the parties herein agreed
to make the premiums payable in installments, and there is no pretense that the
parties never envisioned to make the insurance contract binding between them.
It was renewed for two succeeding years, the second and third policies being a
renewal/replacement for the previous one. And the insured never informed the
insurer that it was terminating the policy because the terms were unacceptable.
Issue: WON, the payment by installment of the premiums due on an insurance
policy invalidates the contract of insurance, in view of Sec. 77 of P.D. 612,
otherwise known as the Insurance Code, as amended.
Held/Ratio: No, while it may be true that under Section 77 of the Insurance
Code(An insurer is entitled to the payment of the premium as soon as the thing is
exposed to the peril insured against. Notwithstanding any agreement to the
contrary, no policy or contract of insurance issued by an insurance company is
valid and binding unless and until the premium thereof has been paid, except in
the case of a life or an industrial life policy whenever the grace period provision

16
applies.), the parties may not agree to make the insurance contract valid and
binding without payment of premiums, there is nothing in said section which
suggests that the parties may not agree to allow payment of the premiums in
installment, or to consider the contract as valid and binding upon payment of the
first premium. Otherwise, we would allow the insurer to renege on its liability
under the contract, had a loss incurred (sic) before completion of payment of the
entire premium, despite its voluntary acceptance of partial payments, a result
eschewed by a basic considerations of fairness and equity.
To our mind, the insurance contract became valid and binding upon payment of
the first premium, and the plaintiff could not have denied liability on the ground
that payment was not made in full, for the reason that it agreed to accept
installment payment. . . .
Tuscany now asserts that its payment by installment of the premiums for the
insurance policies for 1982, 1983 and 1984 invalidated said policies because of
the provisions of Sec. 77 of the Insurance Code, as amended, and by the
conditions stipulated by the insurer in its receipts, disclaiming liability for loss for
occurring before payment of premiums.
It argues that where the premiums is not actually paid in full, the policy would
only be effective if there is an acknowledgment in the policy of the receipt of
premium pursuant to Sec. 78 of the Insurance Code. The absence of an express
acknowledgment in the policies of such receipt of the corresponding premium
payments, and petitioner's failure to pay said premiums on or before the effective
dates of said policies rendered them invalid. Tuscany thus concludes that there
cannot be a perfected contract of insurance upon mere partial payment of the
premiums because under Sec. 77 of the Insurance Code, no contract of insurance
is valid and binding unless the premium thereof has been paid, notwithstanding
any agreement to the contrary. As a consequence, petitioner seeks a refund of all
premium payments made on the alleged invalid insurance policies.
We hold that the subject policies are valid even if the premiums were paid on
installments. The records clearly show that petitioner and private respondent
intended subject insurance policies to be binding and effective notwithstanding
the staggered payment of the premiums. The initial insurance contract entered
into in 1982 was renewed in 1983, then in 1984. In those three (3) years, the
insurer accepted all the installment payments. Such acceptance of payments
speaks loudly of the insurer's intention to honor the policies it issued to petitioner.
Certainly, basic principles of equity and fairness would not allow the insurer to
continue collecting and accepting the premiums, although paid on installments,
and later deny liability on the lame excuse that the premiums were not prepared
in full.

Part 2_Case Digests


3.1_Insurance Law

We therefore sustain the Court of Appeals. We quote with approval the wellreasoned findings and conclusion of the appellate court contained in its
Resolution denying the motion to reconsider its Decision
While the import of Section 77 is that prepayment of premiums is strictly required
as a condition to the validity of the contract, We are not prepared to rule that the
request to make installment payments duly approved by the insurer, would
prevent the entire contract of insurance from going into effect despite payment
and acceptance of the initial premium or first installment. Section 78 of the
Insurance Code in effect allows waiver by the insurer of the condition of
prepayment by making an acknowledgment in the insurance policy of receipt of
premium as conclusive evidence of payment so far as to make the policy binding
despite the fact that premium is actually unpaid. Section 77 merely precludes the
parties from stipulating that the policy is valid even if premiums are not paid, but
does not expressly prohibit an agreement granting credit extension, and such an
agreement is not contrary to morals, good customs, public order or public policy
(De Leon, the Insurance Code, at p. 175). So is an understanding to allow insured
to pay premiums in installments not so proscribed. At the very least, both parties
should be deemed in estoppel to question the arrangement they have voluntarily
accepted.

17
Ambassador the former's lauan round logs numbering 940 at the port of
Maconacon, Isabela for shipment to Manila.

On 20 January 1984, Valenzuela insured the logs, against loss and/or, damage
with defendant South Sea Surety and Insurance Co., Inc. for P2,000,000.00 and
the latter issued its Marine Cargo Insurance Policy No. 84/24229 for
P2,000,000.00 on said date.

On 24 January 1984, the Valenzuela Hardwood gave the check in payment of the
premium on the insurance policy to Mr. Victorio Chua.

On 25 January 1984, M/V Seven Ambassador sank that resulted in the loss of
Valenzuela Hardwoods insured logs.

The reliance by petitioner on Arce vs. Capital Surety and Insurance Co. is
unavailing because the facts therein are substantially different from those in the
case at bar. In Arce, no payment was made by the insured at all despite the grace
period given. In the case before Us, petitioner paid the initial installment and
thereafter made staggered payments resulting in full payment of the 1982 and
1983 insurance policies. For the 1984 policy, petitioner paid two (2) installments
although it refused to pay the balance.

On 30 January 1984, a check for P5,625.00 (Exh. "E") to cover payment of the
premium and documentary stamps due on the policy was tendered to the insurer
but was not accepted. Instead, the South Sea Surety and Insurance Co., Inc.
cancelled the insurance policy it issued as of the date of inception for nonpayment of the premium due in accordance with Section 77 of the Insurance
Code.

It appearing from the peculiar circumstances that the parties actually intended to
make three (3) insurance contracts valid, effective and binding, Tuscany may not
be allowed to renege on its obligation to pay the balance of the premium after the
expiration of the whole term of the third policy (No. AH-CPP-9210651) in March
1985. Moreover, as correctly observed by the appellate court, where the risk is
entire and the contract is indivisible, the insured is not entitled to a refund of the
premiums paid if the insurer was exposed to the risk insured for any period,
however brief or momentary.

On 2 February 1984, Valenzuela Hardwood demanded from defendant South Sea


Surety and Insurance Co., Inc. the payment of the proceeds of the policy but the
latter denied liability under the policy. Valenzuela likewise filed a formal claim
with defendant Seven Brothers Shipping Corporation for the value of the lost logs
but the latter denied the claim.

51) South Sea Surety and Insurance Co. v. Court of Appeals, 244 SCRA
744
Facts: It appears that on 16 January 1984, plaintiff [Valenzuela Hardwood and
Industrial Supply, Inc.] entered into an agreement with the defendant Seven
Brothers whereby the latter undertook to load on board its vessel M/V Seven

The RTC ruled in favor of Valenzuela Hardwood.

The CA affirmed but only as against the insurance corporation (Southsea Surety).
It absolved the shipping entity because of the stipulation in the charter party that

Part 2_Case Digests


3.1_Insurance Law

18

the ship owner (Seven Brothers) would be exempted from liability in case of loss.
Hence, this petition.

the risk insured against has so acted as an agent of petitioner. The appellate
court, like the trial court, has found in the affirmative. Said the appellate court:

Issue: WON, Southsea should also be absolved based on non-payment of


premiums as Victorio Chua acted not its agent. (In other words, was Victorio Chua
its agent? Because he delivered the check for the payment of the premium to
Southsea only AFTER the loss occurred.)

In the instant case, the Marine Cargo Insurance Policy No. 84/24229 was
issued by defendant insurance company on 20 January 1984. At the time the
vessel sank on 25 January 1984 resulting in the loss of the insured logs, the
insured had already delivered to Victorio Chua the check in payment of
premium. But, as Victorio Chua testified, it was only in the morning of 30
January 1984 or 5 days after the vessel sank when his messenger tendered
the check to defendant South Sea Surety and Insurance Co., Inc. (TSN, pp. 327, 16-17, 22 October 1985).

Held/Ratio: Southsea should not be absolved as Chua was its agent. Section 77
of the Insurance Code provides:
Sec. 77. An insurer is entitled to payment of the premium as soon as the
thing insured is exposed to the peril insured against. Notwithstanding any
agreement to the contrary, no policy or contract of insurance issued by an
insurance company is valid and binding unless and until the premium thereof
has been paid, except in the case of a life or an industrial life policy
whenever the grace period provision applies.

Appellant surety company insists that Mr. Chua is an administrative assistant for
the past ten years and an agent for less than ten years of the Columbia Insurance
Brokers, Ltd. He is paid a salary as a administrative assistant and a commission
as agent based on the premiums he turns over to the broker. Southsea therefore
argues that Mr. Chua, having received the insurance premiums as an agent of the
Columbia Insurance Broker, acted as an agent of the insured under Section 301 of
the Insurance Code which provides as follows:

Undoubtedly, the payment of the premium is a condition precedent to, and


essential for, the efficaciousness of the contract. The only two statutorily provided
exceptions are (a) in case the insurance coverage relates to life or industrial life
(health) insurance when a grace period applies and (b) when the insurer makes a
written acknowledgment of the receipt of premium, this acknowledgment being
declared by law to be then conclusive evidence of the premium payment (Secs.
77-78, Insurance Code). The appellate court, contrary to what the petition
suggests, did not make any pronouncement to the contrary. Indeed, it has said:

Sec. 301. Any person who for any compensation, commission or other thing
of value, acts, or aids in soliciting, negotiating or procuring the making of any
insurance contract or in placing risk or taking out insurance, on behalf of an
insured other than himself, shall be an insurance broker within the intent of
this Code, and shall thereby become liable to all the duties requirements,
liabilities and penalties to which an insurance broker is subject.

Concerning the issue as to whether there is a valid contract of insurance


between plaintiff-appellee and defendant-appellant South Sea Surety and
Insurance Co., Inc., Section 77 of the Insurance Code explicitly provides that
notwithstanding any agreement to the contrary, no policy issued by an
insurance company is valid and binding unless and until premium thereof has
been paid. It is therefore important to determine whether at the time of the
loss, the premium was already paid.

Valenzuela Hardwood, upon the other hand, claim that the second paragraph of
Section 306 of the Insurance Code provide as follows:

No attempt becloud the issues can disguise the fact that the sole question raised
in the instant petition is really evidentiary in nature, i.e., whether or not Victorio
Chua, in receiving the check for the insurance premium prior to the occurrence of

Sec. 306. . . . Any insurance company which delivers to an insurance agent or


insurance broker a policy or contract of insurance shall be deemed to have
authorized such agent or broker to receive on its behalf payment of any
premium which is due on such policy of contract of insurance at the time of
its issuance or delivery or which becomes due thereon.

On cross-examination in behalf of South Sea Surety and Insurance Co., Inc. Mr.
Chua testified that the marine cargo insurance policy for the plaintiff's logs was
delivered to him on 21 January 1984 at his office to be delivered to the plaintiff.

Part 2_Case Digests


3.1_Insurance Law
When the appellant South Sea Surety and Insurance Co., Inc. delivered to Mr.
Chua the marine cargo insurance policy for the plaintiffs logs, he is deemed to
have been authorized by the South Sea Surety and Insurance Co., Inc. to receive
the premium which is due on its behalf.

When therefore the insured logs were lost, the insured had already paid the
premium to an agent of the South Sea Surety and Insurance Co., Inc., which is
consequently liable to pay the insurance proceeds under the policy it issued to
the insured.

52) SPS. TIBAY v. COURT OF APPEALS and FORTUNE LIFE AND GENERAL
INSURANCE CO., INC. - Abril
Facts: Jan 22 1987, FORTUNE issued a Fire Insurance Policy in favor of Sps. Tibay
on their 2-storey residential bldg. located in Makati city, together with all their
personal effects therein. The insurance was for P600,000 covering the period
fromJan 23 1987 to Jan 23 1988. On Jan 23 1987, of the total premium of
P2983.50, Violeta Tibay only paid P600 thus leaving a considerable balance
unpaid.
Mar 8 1987, the insured bldg. was completely destroyed by fire. 2 days later (mar
10 1987) Violeta oaid the balance of the premium. On the same day, she filed
with Fortune a claim on the fire insurance policy. The claim was referred to GASI
which immediately wrote Violeta requesting her to furnish it with the necessary
documents for the investigation and processing of her claim. Complied, she
signed (mar 28 1987) a non-waiver agreement with GASI to the effect that any
action taken by the companies or their representatives in investigating the claim
made by the claimant for his loss which occurred at 5855 Zobel Roxas, Makati on
March 8, 1987, or in the investigating or ascertainment of the amount of actual
cash value and loss, shall not waive or invalidate any condition of the policies of
such companies held by said claimant, nor the rights of either or any of the
parties to this agreement, and such action shall not be, or be claimed to be, an
admission of liability on the part of said companies or any of them.
Fortune denied the claim for violation of Policy condition no. 2 and sec. 77 of the
Insurance Code. Efforts to settle the case before the Commission proved futile.
Mar 3 1988, Violeta and the other petitioners sued Fortune in the amount of
P600,000 plus 12% interest per annum, P100,000 moral damages, and attys fees
equivalent to 20% of the total claim.

19
TC adjudged Fortune liable for the total value of the insured bldg. and personal
properties (P600,000).
CA reversed the decision declaring Fortune not liable to the sps. But to return to
the sps the premium plus 12% interest from Mar 10 1987 until full payment.
Issue: Whether or not Fortune remains liable under the fire insurance policy
despite the spouses failure to pay their premium in full.
Ruling: No.
Insurance is a contract whereby one undertakes for a consideration to indemnify
another against loss, damage or liability arising from an unknown or contingent
event. The consideration is the premium, which must be paid at the time and in
the way and manner specified in the policy, and if not so paid, the policy will
lapse and be forfeited by its own terms.
The pertinent provisions in the Policy on premium read:
THIS POLICY OF INSURANCE WITNESSETH, THAT only after payment to the
Company in accordance with Policy Condition No. 2 of the total premiums by the
insured as stipulated above for the period aforementioned for insuring against
Loss or Damage by Fire or Lightning as herein appears, the Property herein
described
xxx
2. This policy including any renewal thereof and/or any endorsement thereon is
not in force until the premium has been fully paid to and duly receipted by the
Company in the manner provided herein.
Any supplementary agreement seeking to amend this condition prepared by
agent, broker or Company official, shall be deemed invalid and of no effect.
xxx
Except only in those specific cases where corresponding rules and regulations
which are or may hereafter be in force provide for the payment of the stipulated
premiums in periodic installments at fixed percentage, it is hereby declared,
agreed and warranted that this policy shall be deemed effective, valid and
binding upon the Company only when the premiums therefor have actually been
paid in full and duly acknowledged in a receipt signed by any authorized official or
representative/agent of the Company in such manner as provided herein, (Italics
supplied)

Part 2_Case Digests


3.1_Insurance Law

Clearly the Policy provides for payment of premium in full. Accordingly, where the
premium has only been partially paid and the balance paid only after the peril
insured against has occurred, the insurance contract did not take effect and the
insured cannot collect at all on the policy. This is fully supported by Sec. 77 of the
Insurance Code which provides
SEC. 77. An insurer is entitled to payment of the premium as soon as the
thing insured is exposed to the peril insured against. Notwithstanding any
agreement to the contrary, no policy or contract of insurance issued by an
insurance company is valid and binding unless and until the premium
thereof has been paid, except in the case of a life or an industrial life
policy whenever the grace period provision applies (Italics supplied).
Apparently the crux of the controversy lies in the phrase unless and until the
premium thereof has been paid. This leads us to the manner of payment
envisioned by the law to make the insurance policy operative and binding. For
whatever judicial construction may be accorded the disputed phrase must
ultimately yield to the clear mandate of the law. The principle that where the law
does not distinguish the court should neither distinguish assumes that the
legislature made no qualification on the use of a general word or expression. In
Escosura v. San Miguel Brewery, inc.,] the Court through Mr. Justice Jesus G.
Barrera, interpreting the phrase with pay used in connection with leaves of
absence with pay granted to employees, ruled x x x the legislative practice seems to be that when the intention is to
distinguish between full and partial payment, the modifying term is used x
xx
Citing C. A. No. 647 governing maternity leaves of married women in government,
R. A. No. 679 regulating employment of women and children, R.A. No. 843
granting vacation and sick leaves to judges of municipal courts and justices of the
peace, and finally, Art. 1695 of the New Civil Code providing that every househelp
shall be allowed four (4) days vacation each month, which laws simply stated with
pay, the Court concluded that it was undisputed that in all these laws the phrase
with pay used without any qualifying adjective meant that the employee was
entitled to full compensation during his leave of absence.
Petitioners maintain otherwise. Insisting that FORTUNE is liable on the policy
despite partial payment of the premium due and the express stipulation thereof
to the contrary, petitioners rely heavily on the 1967 case of Philippine Phoenix
and Insurance Co., Inc. v. Woodworks, Inc. where the Court through Mr. Justice
Arsenio P. Dizon sustained the ruling of the trial court that partial payment of the
premium made the policy effective during the whole period of the policy. In that
case, the insurance company commenced action against the insured for the

20
unpaid balance on a fire insurance policy. In its defense the insured claimed that
nonpayment of premium produced the cancellation of the insurance contract.
Ruling otherwise the Court held
It is clear x x x that on April 1, 1960, Fire Insurance Policy No. 9652 was issued by
appellee and delivered to appellant, and that on September 22 of the same year,
the latter paid to the former the sum of P3,000.00 on account of the total
premium of P6,051.95 due thereon. There is, consequently, no doubt at all that,
as between the insurer and the insured, there was not only a perfected contract
of insurance but a partially performed one as far as the payment of the agreed
premium was concerned. Thereafter the obligation of the insurer to pay the
insured the amount, for which the policy was issued in case the conditions
therefor had been complied with, arose and became binding upon it, while the
obligation of the insured to pay the remainder of the total amount of the premium
due became demandable.
The 1967 Phoenix case is not persuasive; neither is it decisive of the instant
dispute. For one, the factual scenario is different. In Phoenix it was the insurance
company that sued for the balance of the premium, i.e., it recognized and
admitted the existence of an insurance contract with the insured. In the case
before us, there is, quite unlike in Phoenix, a specific stipulation that (t)his policy
xxx is not in force until the premium has been fully paid and duly receipted by
the Company x x x. Resultantly, it is correct to say that in Phoenix a contract was
perfected upon partial payment of the premium since the parties had not
otherwise stipulated that prepayment of the premium in full was a condition
precedent to the existence of a contract.
In Phoenix, by accepting the initial payment of P3,000.00 and then later
demanding the remainder of the premium without any other precondition to its
enforceability as in the instant case, the insurer in effect had shown its intention
to continue with the existing contract of insurance, as in fact it was enforcing its
right to collect premium, or exact specific performance from the insured. This is
not so here. By express agreement of the parties, no vinculum juris or bond of law
was to be established until full payment was effected prior to the occurrence of
the risk insured against.
In Makati Tuscany Condominium Corp. v. Court of Appeals the parties mutually
agreed that the premiums could be paid in installments, which in fact they did for
three (3) years, hence, this Court refused to invalidate the insurance policy. In
giving effect to the policy, the Court quoted with approval the Court of Appeals
The obligation to pay premiums when due is ordinarily an indivisible
obligation to pay the entire premium. Here, the parties x x x agreed to
make the premiums payable in installments, and there is no pretense that

Part 2_Case Digests


3.1_Insurance Law

the parties never envisioned to make the insurance contract binding


between them. It was renewed for two succeeding years, the second and
third policies being a renewal/replacement for the previous one. And the
insured never informed the insurer that it was terminating the policy
because the terms were unacceptable.
While it maybe true that under Section 77 of the Insurance Code, the
parties may not agree to make the insurance contract valid and binding
without payment of premiums, there is nothing in said section which
suggests that the parties may not agree to allow payment of the premiums
in installment, or to consider the contract as valid and binding upon
payment of the first premium. Otherwise we would allow the insurer to
renege on its liability under the contract, had a loss incurred (sic) before
completion of payment of the entire premium, despite its voluntary
acceptance of partial payments, a result eschewed by basic considerations
of fairness and equity x x x.
These two (2) cases, Phoenix and Tuscany, adequately demonstrate the waiver,
either express or implied, of prepayment in full by the insurer: impliedly, by suing
for the balance of the premium as inPhoenix, and expressly, by agreeing to make
premiums payable in installments as in Tuscany. But contrary to the stance taken
by petitioners, there is no waiver express or implied in the case at bench.
Precisely, the insurer and the insured expressly stipulated that (t)his policy
including any renewal thereof and/or any indorsement thereon is not in force until
the premium has been fully paid to and duly receipted by the Company x x x and
that this policy shall be deemed effective, valid and binding upon the Company
only when the premiums therefor have actually been paid in full and duly
acknowledged.
Conformably with the aforesaid stipulations explicitly worded and taken in
conjunction with Sec. 77 of the Insurance Code the payment of partial premium
by the assured in this particular instance should not be considered the payment
required by the law and the stipulation of the parties. Rather, it must be taken in
the concept of a deposit to be held in trust by the insurer until such time that the
full amount has been tendered and duly receipted for. In other words, as
expressly agreed upon in the contract, full payment must be made before the risk
occurs for the policy to be considered effective and in force.
Thus, no vinculum juris whereby the insurer bound itself to indemnify the assured
according to law ever resulted from the fractional payment of premium. The
insurance contract itself expressly provided that the policy would be effective only
when the premium was paid in full. It would have been altogether different were it
not so stipulated. Ergo, petitioners had absolute freedom of choice whether or not
to be insured by FORTUNE under the terms of its policy and they freely opted to

21
adhere thereto.
Indeed, and far more importantly, the cardinal polestar in the construction of an
insurance contract is the intention of the parties as expressed in the policy. Courts
have no other function but to enforce the same. The rule that contracts of
insurance will be construed in favor of the insured and most strongly against the
insurer should not be permitted to have the effect of making a plain agreement
ambiguous and then construe it in favor of the insured. Verily, it is elemental law
that the payment of premium is requisite to keep the policy of insurance in force.
If the premium is not paid in the manner prescribed in the policy as intended by
the parties the policy is ineffective. Partial payment even when accepted as a
partial payment will not keep the policy alive even for such fractional part of the
year as the part payment bears to the whole payment.
Interpreting the contract of insurance stringently against the insurer but liberally
in favor of the insured despite clearly defined obligations of the parties to the
policy can be carried out to extremes that there is the danger that we may, so to
speak, kill the goose that lays the golden egg. We are well aware of insurance
companies falling into the despicable habit of collecting premiums promptly yet
resorting to all kinds of excuses to deny or delay payment of just insurance
claims. But, in this case, the law is manifestly on the side of the insurer. For as
long as the current Insurance Code remains unchanged and partial payment of
premiums is not mentioned at all as among the exceptions provided in Secs. 77
and 78, no policy of insurance can ever pretend to be efficacious or effective until
premium has been fully paid.
And so it must be. For it cannot be disputed that premium is the elixir vitae of the
insurance business because by law the insurer must maintain a legal reserve fund
to meet its contingent obligations to the public, hence, the imperative need for its
prompt payment and full satisfaction. It must be emphasized here that all
actuarial calculations and various tabulations of probabilities of losses under the
risks insured against are based on the sound hypothesis of prompt payment of
premiums. Upon this bedrock insurance firms are enabled to offer the assurance
of security to the public at favorable rates. But once payment of premium is left
to the whim and caprice of the insured, as when the courts tolerate the payment
of a mere P600.00 as partial undertaking out of the stipulated total premium of
P2,983.50 and the balance to be paid even after the risk insured against has
occurred, as petitioners have done in this case, on the principle that the strength
of the vinculum juris is not measured by any specific amount of premium
payment, we will surely wreak havoc on the business and set to naught what has
taken actuarians centuries to devise to arrive at a fair and equitable distribution
of risks and benefits between the insurer and the insured.

Part 2_Case Digests


3.1_Insurance Law

The terms of the insurance policy constitute the measure of the insurers liability.
In the absence of statutory prohibition to the contrary, insurance companies have
the same rights as individuals to limit their liability and to impose whatever
conditions they deem best upon their obligations not inconsistent with public
policy. The validity of these limitations is by law passed upon by the Insurance
Commissioner who is empowered to approve all forms of policies, certificates or
contracts of insurance which insurers intend to issue or deliver. That the policy
contract in the case at bench was approved and allowed issuance simply
reaffirms the validity of such policy, particularly the provision in question.

53) PAULIN v. INSULAR


Tried my best to look for this case, but unfortunately, cannot be found in the net
as well as in CD Asia.

22
renewed, and (b) that the fire occurred on Jun 13 1992, before Masaganas tender
of premium payment.
Jul 21 1992, Masagana filed with RTC against UCPB for recover of P18,645,000,
representing the face value of the policies,
Oct 23 1992, motion to dismiss denied, filed an answer alleging that the
complaint fails to state a cause of action; that UCPB was not liabel to Masagana
for insurance proceeds under the policies because at the time of the loss of the
property due to fire, the policies had long expired and were not renewed.
RTC: ruled in favor of Masagana, allowing it to consign with the court full payment
of the corresponding premiums, declaring Masagana to have fully complied with
its obligation to pay the premium thereby rendering the replacement-renewal
policy, and ordering UCPB to pay the sums of P18,645,000, 25% of the total
amount due as and for attys fees, and P25,000 for litigation expenses and the
costs of suit.

54) UCPB GENERAL INSURANCE CO. v. MASAGANA TELEMART (1999)


Facts: Apr 15 1991, UCPB issued 5 insurance policies covering Masaganas
various properties against fire, for the period from May 22 1991 to May 22 1992.
March 1992, UCPB evaluated the policies and decided not to renew them upon
expiration of their terms on May 22 1992. UCPB advised Masaganas broker,
Zuellig of its intention not to renew the policies.
Apr 6 1992. UCPB gave written notice to Masagana of the non-renewal of the
policies at the address stated in the policies.
Jun 13 1992, fire razed Masaganas property covered by 3 of the insurance
policies UCPB issued.
Jul 13 1992, Masagana presented to UCPBs cashier 5 managers checks in the
total amount of P225,753.95, representing premium for the renewal of the
policies from May 22 1992 to May 22 1993. No notice of loss was filed by
Masagana under the policies prior to Jul 14 1992.
Jul 14 1992, Masagana filed with UCPB its formal claim for indemnification of the
insured property.
On the same day, UCPB retuned to Masagana the 5 checks that it tendered, and
rejected its claim for the reasons (a) that the policies had expired and were not

CA: affirmed the decision of the RTC, held that Masagana was allowed a 60 to 90
day credit term for the renewal of its policies, and that the acceptance of the late
premium payment suggested an understanding that payment could be made
later.
Issue: Whether or not the fire insurance policies had expired on the May 22 1992
or had been extended or renewed by an implied credit arrangement through
actual payment of premium was tendered on a later date after the occurrence of
the fire insured against.
Ruling: They expired and were not extended.
The answer is easily found in the Insurance Code. No, an insurance policy, other
than life, issued originally or on renewal, is not valid and binding until actual
payment of the premium. Any agreement to the contrary is void. The parties may
not agree expressly or impliedly on the extension of credit or time to pay the
premium and consider the policy binding before actual payment.
The case of Malayan Insurance Co., Inc. vs. Cruz-Arnaldo ciited by the Court of
Appeals, is not applicable. In that case, payment of the premium was in fact
actually made on December 24, 1981, and the fire occurred on January 18, 1982.
Here, the payment of the premium for renewal of the policies was tendered on
July 13, 1992, a month after the fire occurred on June 13, 1992. The assured
did not even give the insurer a notice of loss within a reasonable time after
occurrence of the fire.

Part 2_Case Digests


3.1_Insurance Law

23

Part 2_Case Digests


3.1_Insurance Law

55) UCPB GENERAL INSURANCE CO. v. MASAGANA TELEMART (2001)


Facts: The SC in its Jue 15 1999 decisions reversed and set aside the assailed
decision of the CA which affirmed with modification the judgment of the RTC (a)
allowing Masagana to consign the sum of P225,735.95 as full payment of the
premiums for the renewal of the 5 insurance policies on its properties; (b)
declaring the replacement-renewal policies effective and binding from May 22
1992 until May 22 1993; and (c) ordering UCPB to pay Masagana indemnity for
the said properties.
Operative facts:
All five (5) policies reflect on their face the effectivity term: "from 4:00 P.M. of 22
May 1991 to 4:00 P.M. of 22 May 1992." On June 13, 1992, plaintiff's properties
located at 2410-2432 and 2442-2450 Taft Avenue, Pasay City were razed by fire.
On July 13, 1992, plaintiff tendered, and defendant accepted, five (5) Equitable
Bank Manager's Checks in the total amount of P225,753.45 as renewal premium
payments for which Official Receipt Direct Premium No. 62926 (Exhibit "Q",
Record, p. 191) was issued by defendant. On July 14, 1992, Masagana made its
formal demand for indemnification for the burned insured properties. On the
same day, defendant returned the five (5) manager's checks stating in its letter
(Exhibit "R"/"8", Record, p. 192) that it was rejecting Masagana's claim on the
following grounds:
"a) Said policies expired last May 22, 1992 and were not renewed for
another term;
b) Defendant had put plaintiff and its alleged broker on notice of nonrenewal earlier; and
c) The properties covered by the said policies were burned in a fire that
took place last June 13, 1992, or before tender of premium payment."
The Court of Appeals disagreed with Petitioners stand that Respondents tender of
payment of the premiums on 13 July 1992 did not result in the renewal of the
policies, having been made beyond the effective date of renewal as provided
under Policy Condition No. 26, which states:
26. Renewal Clause. -- Unless the company at least forty five days in
advance of the end of the policy period mails or delivers to the assured at
the address shown in the policy notice of its intention not to renew the
policy or to condition its renewal upon reduction of limits or elimination of
coverages, the assured shall be entitled to renew the policy upon payment
of the premium due on the effective date of renewal.

24
Both the Court of Appeals and the trial court found that sufficient proof exists that
Masagana, which had procured insurance coverage from UCPB for a number of
years, had been granted a 60 to 90-day credit term for the renewal of the
policies. Such a practice had existed up to the time the claims were filed.
In the SCs 1999 Decision it defined the main issue whether the fire insurance
policies issued by UCPB to Masagana covering the period had been extended or
renewed by an implied credit arrangement thru actual payment of premium was
tendered on a later date and after the occurrence of the fire insured against. The
SC ruled in the negative, and accordingly set aside and reversed the decision of
the CA.
Masagana filed a MR: that the SC made in the decision it own findings of facts,
which are not in accord with those of the TC and the CA. The courts below
correctly found that no notice of non-renewal was made within 45 days before
May 22 1992, or before the expiration date of the fire insurance policies. Thus, the
same were renewed by operation of law thus were effective and valid on Jun 30
1992 when the fire occurred.
Masagana also disagrees that parties may neither agree expressly or impliedly on
the extension of credit or time to pay the premium nor consider a policy binding
before actual payment; that despite the express provision of sec. 77 of the IC,
extension of credit terms in premium payment has been the prevalent practice in
the insurance industry; that extension of credit terms is not a prohibitive
injunction but merely designed for the protection of parties to an insurance
contract.
UCPB opposition: both the TC and CA overlooked the fact that on Apr 6 1992
UCPB sent by ordinary mail to Masagana a notice of non-renewal and sent by
personal delivery a copy thereof to Masaganas broker, Zuellig; that the courts
ignored the fact that Masagana was fully aware of the notice of non-renewal; that
a reading of Section 66 of the Insurance Code readily shows that in order for an
insured to be entitled to a renewal of a non-life policy, payment of the premium
due on the effective date of renewal should first be made, thus, Masaganas
argument that Section 77 is not a prohibitive provision finds no authoritative
support.
Upon a meticulous review of the records and reevaluation of the issues raised in
the motion for reconsideration and the pleadings filed thereafter by the parties,
we resolved to grant the motion for reconsideration. The following facts, as found
by the trial court and the Court of Appeals, are indeed duly established:
1. For years, Petitioner had been issuing fire policies to the Respondent, and
these policies were annually renewed.

Part 2_Case Digests


3.1_Insurance Law

2. Petitioner had been granting Respondent a 60- to 90-day credit term


within which to pay the premiums on the renewed policies.
3. There was no valid notice of non-renewal of the policies in question, as
there is no proof at all that the notice sent by ordinary mail was received by
Respondent, and the copy thereof allegedly sent to Zuellig was ever
transmitted to Respondent.
4. The premiums for the policies in question in the aggregate amount of
P225,753.95 were paid by Respondent within the 60- to 90-day credit term
and were duly accepted and received by Petitioners cashier.
Issue: Whether or not sec. 77 of the Insurance Code of 1978 must be strictly
applied to UCPBs advantage despite its practice of granting a 60-90 day credit
term for the payment of premiums.
Ruling: Section 77 of the Insurance Code of 1978 provides:
SEC. 77. An insurer is entitled to payment of the premium as soon as the
thing insured is exposed to the peril insured against. Notwithstanding any
agreement to the contrary, no policy or contract of insurance issued by an
insurance company is valid and binding unless and until the premium
thereof has been paid, except in the case of a life or an industrial life
policy whenever the grace period provision applies.
This Section is a reproduction of Section 77 of P.D. No. 612 (The Insurance
Code) promulgated on 18 December 1974. In turn, this Section has its source in
Section 72 of Act No. 2427 otherwise known as the Insurance Act as amended by
R.A. No. 3540, approved on 21 June 1963, which read:
SEC. 72. An insurer is entitled to payment of premium as soon as the thing
insured is exposed to the peril insured against, unless there is clear
agreement to grant the insured credit extension of the premium due. No
policy issued by an insurance company is valid and binding unless and until
the premium thereof has been paid. (Underscoring supplied)
It can be seen at once that Section 77 does not restate the portion of Section 72
expressly permitting an agreement to extend the period to pay the premium. But
are there exceptions to Section 77?
The answer is in the affirmative.
The first exception is provided by Section 77 itself, and that is, in case of a life or
industrial life policy whenever the grace period provision applies.

25
The second is that covered by Section 78 of the Insurance Code, which provides:
SEC. 78. Any acknowledgment in a policy or contract of insurance of the
receipt of premium is conclusive evidence of its payment, so far as to make
the policy binding, notwithstanding any stipulation therein that it shall not
be binding until
A third exception was laid down in Makati Tuscany Condominium Corporation vs.
Court of Appeals, wherein we ruled that Section 77 may not apply if the parties
have agreed to the payment in installments of the premium and partial payment
has been made at the time of loss. We said therein, thus:
We hold that the subject policies are valid even if the premiums were paid
on installments. The records clearly show that the petitioners and private
respondent intended subject insurance policies to be binding and effective
notwithstanding the staggered payment of the premiums. The initial
insurance contract entered into in 1982 was renewed in 1983, then in
1984. In those three years, the insurer accepted all the installment
payments. Such acceptance of payments speaks loudly of the insurers
intention to honor the policies it issued to petitioner. Certainly, basic
principles of equity and fairness would not allow the insurer to continue
collecting and accepting the premiums, although paid on installments, and
later deny liability on the lame excuse that the premiums were not prepaid
in full.
By the approval of the aforequoted findings and conclusion of the Court of
Appeals, Tuscany has provided a fourth exception to Section 77, namely, that the
insurer may grant credit extension for the payment of the premium. This simply
means that if the insurer has granted the insured a credit term for the payment of
the premium and loss occurs before the expiration of the term, recovery on the
policy should be allowed even though the premium is paid after the loss but
within the credit term.
Moreover, there is nothing in Section 77 which prohibits the parties in an
insurance contract to provide a credit term within which to pay the premiums.
That agreement is not against the law, morals, good customs, public order or
public policy. The agreement binds the parties. Article 1306 of the Civil Code
provides:
ART. 1306. The contracting parties may establish such stipulations clauses,
terms and conditions as they may deem convenient, provided they are not
contrary to law, morals, good customs, public order, or public policy.
Finally in the instant case, it would be unjust and inequitable if recovery on the
policy would not be permitted against UCPB, which had consistently granted a 60-

Part 2_Case Digests


3.1_Insurance Law

to 90-day credit term for the payment of premiums despite its full awareness of
Section 77. Estoppel bars it from taking refuge under said Section, since
Respondent relied in good faith on such practice. Estoppel then is the fifth
exception to Section 77.

26

Part 2_Case Digests


3.1_Insurance Law

56) AMERICAN HOME ASSURANCE v. CHUA


Facts: 1990, Chua obtained from AHA a fire insurance covering the stock-in-trade
of his business, Moonlight Enterprises, located in Bukidnon. The insurance was
due to expire on Mar 25 1990.
Apr 5 1990: Chua issued PCI bank check in the amt of P2,893.50 to AHAs agent,
James Uy, as payment for the renewal of the policy. In turn, AHA delivered the
Renewal Certificate to Chua. The check was drawn against a Manila bank and
deposited in AHAs bank acct in CDO City. Subsequently, a new insurance policy
was issued whereby AHA undertook to indemnify Chua for any damage or loss
arising from fire up to P200,000 (Mar 25 1990 Mar 25 1991).
Apr 6 1990, Moonlight Enterprises was completely razed by fire; total loss P4M
P5M. Chua filed insurance claim with AHA and 4 other co-insurers (Pioneer,
Prudential, Filipino Merchants, and Domestic Insurance). AHA refused to the claim,
hence, action against it before the trial court.
AHA, in its defense, claimed that there was no existing insurance contract when
the fire occurred since Chua did not pay the premium; that even assuming there
was a contract, Chua violated several conditions of the policy, particularly: (1) his
submission of fraudulent income tax return and financial statements, (2) failure to
establish actual loss, which AHA assessed at P70,000, and (3) his failure to notify
to AHA of any insurance already effected to cover the insured goods.
TC: ruled in favor of Chua; held that Chua paid by way of check a day before the
fire occurred, deposited in AHAs account and the same was even acknowledged
in the renewal certificate issued by AHAs agent; that the alleged fraudulent
documents were limited to the disparity between the official receipts issued by
the BIR and the ITRs for the years 1987 to 1989; that all other documents were
found to be genuine; that the failure of Chua to notify AHA of the other insurance
contracts were not intentional nor fraudulent.
CA: affirmed the decision in toto; that Chuas claim was substantially proved and
AHAs unjustified refusal to pay the claim entitled Chua to the award of damages.
American Home filed the petition reiterating its stand that there was no existing
insurance contract between the parties. It invoked Section 77 of the Insurance
Code, which provides that no policy or contract of insurance issued by an
insurance company is valid and binding unless and until the premium thereof has
been paid and the case of Arce v. Capital Insurance that until the premium is paid
there is no insurance.
Issues:

27
a. Whether or not there was a valid payment of premium, considering that
Chuas check was cashed after the fire.
b. Whether or not Chua violated the policy by his submission of fraudulent
documents and non-disclosure of other insurance contracts
Ruling:
a. Yes.
The
renewal certificate
issued to respondent contained the
acknowledgment that premium had been paid. It is not disputed that the
check drawn by respondent in favor of petitioner and delivered to its agent
was honored when presented and petitioner forthwith issued its official
receipt to respondent on 10 April 1990. Section 306 of the Insurance Code
provides that any insurance company which delivers a policy or contract of
insurance to an insurance agent or insurance broker shall be deemed to
have authorized such agent or broker to receive on its behalf payment of
any premium which is due on such policy or contract of insurance at the
time of its issuance or delivery or which becomes due thereon. In the
instant case, the best evidence of such authority is the fact that petitioner
accepted the check and issued the official receipt for the payment. It is, as
well, bound by its agents acknowledgment of receipt of payment.
In the instant case, the best evidence of such authority is the fact that
petitioner accepted the check and issued the official receipt for the
payment. It is, as well, bound by its agents acknowledgment of receipt of
payment.
Section 78 of the Insurance Code explicitly provides:
An acknowledgment in a policy or contract of insurance of the receipt of
premium is conclusive evidence of its payment, so far as to make the
policy binding, notwithstanding any stipulation therein that it shall not be
binding until the premium is actually paid.
b. No.
Submission of the alleged fraudulent documents pertained to respondents
income tax returns for 1987 to 1989. Respondent, however, presented a
BIR certification that he had paid the proper taxes for the said years.
Since this is a question of fact, the finding is conclusive.
Ordinarily, where the insurance policy specifies as a condition the
disclosure of existing co-insurers, non-disclosure is a violation that entitles

Part 2_Case Digests


3.1_Insurance Law

the insurer to avoid the policy. The purpose for the inclusion of this clause
is to prevent an increase in the moral hazard. The relevant provision is
Section 75, which provides that:

28
At the time of the insurance, an insurance policy issued by the Great American
Insurance Company covering the same properties was noted on said policy as coinsurance. Later, the parties executed an endorsement on the policy, stating:

A policy may declare that a violation of specified provisions thereof


shall avoid it, otherwise the breach of an immaterial provision does
not avoid the policy.

It is hereby declared and agreed that the co-insurance existing at


present under this policy is as follows: P20,000.00 Northwest
Ins., and not as originally stated. (emphasis supplied)

Respondent acquired several co-insurers and he failed to disclose this


information to petitioner.
Nonetheless, petitioner is estopped from
invoking this argument due to the loss adjusters admission of previous
knowledge of the co-insurers.

Except as varied by this endorsement, all other terms and


conditions remain unchanged.

It cannot be said that petitioner was deceived by respondent by the


latters non-disclosure of the other insurance contracts when petitioner
actually had prior knowledge thereof. The loss adjuster, being an
employee of petitioner, is deemed a representative of the latter whose
awareness of the other insurance contracts binds petitioner.

K. Double Insurance - Cebrecus


57) PIONEER INSURANCE AND SURETY CORPORATION VS OLIVA YAP, G.R.
NO. L-36232
Facts: Respondent Oliva Yap was the owner of a store in a 2 storey building
where she sold shopping bags and footwear. Chua Soon Poon, Oliva Yaps son-inlaw, was in charge of the store. Respondent Yap took out a fire insurance policy
from petitioner Pioneer insurance and surety corporation. Among the conditions in
the policy executed by the parties are the following:
The Insured shall give notice to the Company of any insurance or
insurances already effected, or which may subsequently be
effected, covering any of the property hereby insured, and unless
such notice be given and the particulars of such insurance or
insurances be stated in, or endorsed on this Policy by or on behalf
of the Company before the occurrence of any loss or damage, all
benefits under this Policy shall be forfeited. (emphasis supplied)
It is understood that, except as may be stated on the face of this
policy there is no other insurance on the property hereby covered
and no other insurance is allowed except by the consent of the
Company endorsed hereon. Any false declaration or breach or this
condition will render this policy null and void.

Still later, Oliva Yap took out another fire insurance policy covering the same
properties, this time from Federal Insurance Company, which new policy was,
however, procured without notice to and the written consent of petitioner Pioneer
Insurance & Surety Corporation and therefore, was not noted as a co-insurance on
the policy.
Later, a fire broke out in the building housing Yaps store and the said store was
burned. Respondent Yap filed an insurance claim but the same was denied on the
ground of breach and/or violation of any and/or terms and conditions of the
policy.
Oliva Yap filed the present complaint.
Issue: WON petitioner should be absolved from liability on fire insurance policy
on account of any violation by respondent Yap of the co-insurance clause. Yes
Held: There was a violation by respondent Oliva Yap of the co-insurance clause
contained in Policy No. 4219 that resulted in the avoidance of petitioner's liability.
The insurance policy for P20,000.00 issued by the Great American Insurance
Company covering the same properties of respondent Yap and duly noted on
Policy No. 4219 as c-insurance, ceased, by agreement of the parties (Exhibit "1L"), to be recognized by them as a co-insurance policy. The Court of Appeals says
that the Great American Insurance policy was substituted by the Federal
Insurance policy for the same amount, and because it was a mere case of
substitution, there was no necessity for its endorsement on Policy No. 4219. This
finding, as well as reasoning, suffers from several flaws. There is no evidence to
establish and prove such a substitution. If anything was substituted for the Great
American Insurance policy, it could only be the Northwest Insurance policy for the
same amount of P20,000.00. The endorsement (Exhibit "1-K") quoted above
shows the clear intention of the parties to recognize on the date the endorsement
was made (August 29, 1962), the existence of only one co-insurance, and that is
the Northwest Insurance policy, which according to the stipulation of the parties
during the hearing, was issued on August 20, 1962 (t.s.n., January 12, 1965, pp.
3-4) and endorsed only on August 20, 1962. The finding of the Court of Appeals

Part 2_Case Digests


3.1_Insurance Law

that the Great American Insurance policy was substituted by the Federal
Insurance policy is unsubstantiated by the evidence of record and indeed contrary
to said stipulation and admission of respondent, and is grounded entirely on
speculation, surmises or conjectures, hence, not binding on the Supreme Court.
The Court of Appeals would consider petitioner to have waived the formal
requirement of endorsing the policy of co-insurance "since there was absolutely
no showing that it was not aware of said substitution and preferred to continue
the policy." The fallacy of this argument is that, contrary to Section 1, Rule 131 of
the Revised Rules of Court, which requires each party to prove his own
allegations, it would shift to petitioner, respondent's burden of proving her
proposition that petitioner was aware of the alleged substitution, and with such
knowledge preferred to continue the policy. Respondent Yap cites Gonzales La O
vs. Yek Tong Lin Fire and Marine Insurance Co., Ltd. to justify the assumption but
in that case, unlike here, there was knowledge by the insurer of violations of the
contract, to wit: "If, with the knowledge of the existence of other insurances which
the defendant deemed violations of the contract, it has preferred to continue the
policy, its action amounts to a waiver of the annulment of the contract ..." A
waiver must be express. If it is to be implied from conduct mainly, said conduct
must be clearly indicative of a clear intent to waive such right. Especially in the
case at bar where petitioner is assumed to have waived a valuable right, nothing
less than a clear, positive waiver, made with full knowledge of the circumstances,
must be required.
By the plain terms of the policy, other insurance without the consent of
petitioner would ipso facto avoid the contract. It required no affirmative
act of election on the part of the company to make operative the clause
avoiding the contract, wherever the specified conditions should occur.
Its obligations ceased, unless, being informed of the fact, it consented
to the additional insurance.
The validity of a clause in a fire insurance policy to the effect that the
procurement of additional insurance without the consent of the insurer renders
ipso facto the policy void is well-settled: In Milwaukee Mechanids' Lumber Co., vs.
Gibson, 199 Ark. 542, 134 S. W. 2d 521, 522, a substantially identical clause was
sustained and enforced, the court saying: "The rule in this state and practically all
of the states is to the effect that a clause in a policy to the effect that the
procurement of additional insurance without the consent of the insurer renders
the policy void is a valid provision. The earlier cases of Planters Mutual Insurance
Co., vs. Green, 72 Ark. 305, 80 S.W. 92, are to the same effect." And see Vance,
Insurance, 2nd Ed., 725. (Reach vs. Arkansas Farmers Mut. Fire Ins. Co., [Ark. Nov.
14, 1949] 224 S. W. 2d 48, 49.)
The annotation then, must be deemed to be a warranty that the

29
property was not insured by any other policy. Violation thereof entitled
the insurer to rescind. (Sec. 69, Insurance Act.) Such misrepresentation
is fatal in the light of our views in Santa Ana vs. Commercial Union
Assurance Company, Ltd., 55 Phil. 329. The materiality of non-disclosure
of other insurance policies is not open to doubt.
Furthermore, even if the annotations were overlooked the defendant insurer
would still be free from liability because there is no question that the policy issued
by General Indemnity has not been stated in nor endorsed on Policy No. 471 of
defendant. And as stipulated in the above-quoted provisions of such policy "all
benefit under this policy shall be forfeited. (Emphasis supplied)
The obvious purpose of the aforesaid requirement in the policy is to
prevent over-insurance and thus avert the perpetration of fraud. The
public, as well as the insurer, is interested in preventing the situation in which a
fire would be profitable to the insured. According to Justice Story: "The insured has
no right to complain, for he assents to comply with all the stipulation on his side,
in order to entitle himself to the benefit of the contract, which, upon reason or
principle, he has no right to ask the court to dispense with the performance of his
own part of the agreement, and yet to bind the other party to obligations, which,
but for those stipulation would not have been entered into."
In view of the above conclusion, We deem it unnecessary to consider the other
defenses interposed by petitioner.

58) UNION MANUFACTURING CO., INC AND THE REPUBLIC BANK VS


PHILIPPINE GUARANTY CO., INC., G.R. NO. L-27932, OCTOBER 30, 1972
Facts: On January 12, 1962, Union Manufacturing obtained certain loans,
overdrafts and other credit accommodations from Republic bank and to secure
payment thereof, said Union Manufacturing executed a real and chattel
mortgages on certain properties, which are more particularly described and listed
at the back of the mortgage contract. That as additional condition of the
mortgage contract, Union Manufacturing undertook to secure insurance coverage
over the mortgaged properties.
Union manufacturing failed to secure insurance coverage on the mortgaged
properties despite the fact that Cua Tok, its general manager, was reminded of
said requirement, Republic Bank procured from Philippine Guaranty an insurance
coverage on loss against fire over the properties of Union Manufacturing as
described in Philippine Guarantys Cover Note with the annotation that the loss
or damage, if any, under said cover note, is payable to republic bank as its
interest may appear, subject however to the printed conditions of said

Part 2_Case Digests


3.1_Insurance Law

defendants fire insurance policy form.


A fire insurance policy was issued in favor of assured, Union Manufacturing for
which a premium was paid by the republic bank to Phil. Guaranty. It appears that
although said renewal premium was paid by the Republic Bank, such payment
was for the account of Union Manufacturing and that the cash voucher for the
payment of the first premium was paid also by the Republic Bank but for the
account of Union Manufacturing.
A fire occurred in the premises of Union Manufacturing. They then filed a fire
claim with the Philippine Guaranty which was denied on the ground that: Policy
condition no. 3 and/or the Other Insurance Clause of the policy violated because
you did not give notice to us the other insurance which you had taken from New
India and Manila Insurance with the results that these insurances, of which we
became aware of only after the fire, were not endorsed on our policy; and Policy
condition no. 1 was not complied with because you have failed failed to give to
our representatives the required documents and other proofs with respect to your
claim and matters touching on our liability, if any, and the amount of such
liability.
That when the defendant Philippine Guaranty issued a fire insurance policy to
cover the properties of the Union Manufacturing Co., the same properties were
already covered by fire policy of the Sincere insurance company and Oceanic
Insurance. That when said Philippine Guarantys fire insurance policy was already
in full force and effect, the Union Manufacturing without the consent of the
Philippine Guaranty, obtained another insurance policy over the same property
prior to the fire from New india Assurance, Sincere Insurance company and Manila
Insurance Co.
Issue: WON Republic Bank can recover from the Philippine Guaranty No
Held: Why the appellant Republic Bank could not recover, as payee, in case of
loss as its "interest may appear subject to the terms and conditions, clauses and
warranties" of the policy was expressed in the appealed decision thus: "However,
inasmuch as the Union Manufacturing Co., Inc. has violated the condition of
the policy to the effect that it did not reveal the existence of other
insurance policies over the same properties, as required by the warranty
appearing on the face of the policy issued by the defendant and that on the other
hand said Union Manufacturing Co., Inc. represented that there were no
other insurance policies at the time of the issuance of said defendant's
policy, and it appearing furthermore that while the policy of the defendant
was in full force and effect the Union Manufacturing Co., Inc. secured
other fire insurance policies without the written consent of the
defendant endorsed on the policy, the conclusion is inevitable that both the

30
Republic Bank and Union Manufacturing Co., Inc. cannot recover from
the same policy of the defendant because the same is null and void." The
tone of confidence apparent in the above excerpts from the lower court decision
is understandable. The conclusion reached by the lower court finds support in
authoritative precedents. It is far from easy, therefore, for appellant Republic
Bank to impute to such a decision a failure to abide by the law. Hence, as noted at
the outset, the appeal cannot prosper. An affirmance is indicated.
It is to Santa Ana v. Commercial Union Assurance Co., a 1930 decision, that one
turns to for the first explicit formulation as to the controlling principle. As was
made clear in the opinion of this Court, penned by Justice Villa-Real: "Without
deciding whether notice of other insurance upon the same property must be
given in writing, or whether a verbal notice is sufficient to render an insurance
valid which requires such notice, whether oral or written, we hold that in the
absolute absence of such notice when it is one of the conditions
specified in the fire insurance policy, the policy is null and void." The next
year, in Ang Giok Chip v. Springfield Fire & Marine Ins. Co., the conformity of the
insured to the terms of the policy, implied from the failure to express any
disagreement with what is provided for, was stressed in these words of the
ponente, Justice Malcolm: "It is admitted that the policy before us was accepted
by the plaintiff. The receipt of this policy by the insured without objection binds
both the acceptor and the insured to the terms thereof. The insured may not
thereafter be heard to say that he did not read the policy or know its terms, since
it is his duty to read his policy and it will be assumed that he did so. As far back
as 1915, in Young v. Midland Textile Insurance Company, it was categorically set
forth that as a condition precedent to the right of recovery, there must be
compliance on the part of the insured with the terms of the policy. As stated in the
opinion of the Court through Justice Johnson: "If the insured has violated or failed
to perform the conditions of the contract, and such a violation or want of
performance has not been waived by the insurer, then the insured cannot recover.
Courts are not permitted to make contracts for the parties. The function and duty
of the courts consist simply in enforcing and carrying out the contracts actually
made. While it is true, as a general rule, that contracts of insurance are construed
most favorably to the insured, yet contracts of insurance, like other contracts, are
to be construed according to the sense and meaning of the terms which the
parties themselves have used. If such terms are clear and unambiguous they
must be taken and understood in their plain, ordinary and popular sense." More
specifically, there was a reiteration of this Santa Ana ruling in a decision by the
then Justice, later Chief Justice, Bengzon, in General Insurance & Surety Corp. v.
Ng Hua. 12 Thus: "The annotation then, must be deemed to be a warranty that the
property was not insured by any other policy. Violation thereof entitles the insurer
to rescind. (Sec. 69, Insurance Act) Such misrepresentation is fatal in the light of
our views in Santa Ana v. Commercial Union Assurance Company, Ltd. ... . The
materiality of non-disclosure of other insurance policies is not open to doubt. As

Part 2_Case Digests


3.1_Insurance Law

a matter of fact, in a 1966 decision, Misamis Lumber Corp. v. Capital Ins. & Surety
Co., Inc., Justice J.B.L. Reyes, for this Court, made manifest anew its adherence to
such a principle in the face of an assertion that thereby a highly unfavorable
provision for the insured would be accorded recognition. This is the language
used: "The insurance contract may be rather onerous ('one sided', as the lower
court put it), but that in itself does not justify the abrogation of its express terms,
terms which the insured accepted or adhered to and which is the law between the
contracting parties.
There is no escaping the conclusion then that the lower court could not have
disposed of this case in a way other than it did. Had it acted otherwise, it clearly
would have disregarded pronouncements of this Court, the compelling force of
which cannot be denied. There is, to repeat, no justification for a reversal.

31

Part 2_Case Digests


3.1_Insurance Law

L. Marine Insurance Cebrecus, as modified


59) ORIENTAL ASSURANCE CORPORATION VS COURT OF APPEALS AND
PANAMA SAWMILL,
G.R. NO. 94052, AUGUST 9, 1991
Facts: Panama Sawmill bought 1,208 pieces of apitong log which it transported
by sea to Manila through Transpacific towage and insured it against loss with
Oriental Assurance corporation. Oriental assurance issued a marine insurance
policy which stated, among others, that the insurance warranted against total loss
only.
The 2 barges were towed by 1 tug-boat but during the voyage, rough seas and
strong winds caused damage to the barge resulting in the total loss of 497 pieces
of logs out of the 598 pieces loaded thereon. Panama demanded payment for the
loss but Oriental assurance refused on the ground that its contracted liability was
for total loss only. The rejection was upon the recommendation of the Tan Gatue
Adjustment Company.
Panama filed a Complaint for Damages against Oriental Assurance et al. with RTC.
Issue: WON Oriental Assurance can be held liable under its marine insurance
policy based on the theory of a divisible contract of insurance and, consequently,
a constructive total loss.
Held: No, as no liability attaches. In the absence of either actual or
constructive total loss, there can be no recovery by the insured Panama
against the insurer, Oriental Assurance.
The terms of the contract constitute the measure of the insurers liability and
compliance therewith is a condition precedent to the insured's right to recovery
from the insurer. Whether a contract is entire or severable is a question of
intention to be determined by the language employed by the parties. The
policy in question shows that the subject matter insured was the entire shipment
of 2,000 cubic meters of apitong logs. Only one premium was paid for the
entire shipment, making for only one cause or consideration. The
insurance contract must, therefore, be considered indivisible.

32
(d) Any other event which effectively deprives the owner of the
possession, at the port of destination, of the thing insured.
A constructive total loss is one which gives to a person insured a right to
abandon, under Section 139 of the Insurance Code. This provision reads:
A person insured by a contract of marine insurance may abandon the
thing insured, or any particular portion thereof separately valued by
the policy, or otherwise separately insured, and recover for a total loss
thereof, when the cause of the loss is a peril injured against, (a) If
more than three-fourths thereof in value is actually lost, or would have
to be expended to recover it from the peril; (b) If it is injured to such an
extent as to reduce its value more than three-fourths xxx
The CA treated the loss as a constructive total loss, and for the purpose of
computing the more than three-fourths value of the logs actually lost, considered
the cargo in one barge as separate from the logs in the other. Thus, it concluded
that the loss of 497 pieces of logs from barge TPAC-1000, mathematically
speaking, is more than three-fourths () of the 598 pieces of logs loaded in that
barge and may, therefore, be considered as constructive total loss. Erroneous.
The requirements for the application of Section 139 of the Insurance Code, have
not been met. The logs involved, although placed in two barges, were not
separately valued by the policy, nor separately insured. Resultantly, the
logs lost in barge TPAC-1000 in relation to the total number of logs loaded therein
on the same barge cannot be made the basis for determining constructive total
loss. The logs having been insured as one inseparable unit, the correct basis for
determining the constructive total loss is the totality of the shipment of logs. So,
out of the 1,208, pieces of logs, only 497 pieces were lost or 41.45% of the entire
shipment. Since it is less than 75% of the value of all 1,208 pieces of logs, the
shipment cannot be said to have sustained a constructive total loss.

More importantly, the insurer's liability was for "total loss only." A total loss may
be either actual or constructive. An actual total loss is caused by:

60) ISABELA ROQUE AND ONG CHIONG VS HON. INTERMEDIATE


APPELLATE
COURT
AND
PIONEER
INSURANCE
AND
SURETY
CORPORATION, G.R. NO. L-66935, NOVEMBER 11, 1985

(a) A total destruction of the thing insured;


(b) The irretrievable loss of the thing by sinking, or by being broken up;
(c) Any damage to the thing which renders it valueless to the owner for
the purpose for which he held it; or

Facts: Manila Bay Lighterage Corporation, a common carrier, entered into a


contract with the petitioners whereby the former would load and carry on board
logs in its barge Marble 10. The petitioners insured the logs against loss with
Pioneer Insurance and Surety Corporation.

Part 2_Case Digests


3.1_Insurance Law

However, the shipment of pieces of logs never reached its destination because
the barge sank. As found by both the trial court and appellate courts, the barge
where the logs were loaded was not seaworthy such that it developed a leak. The
appellate court further found that one of the hatches was left open causing water
to enter the barge and because the barge was not provided with the necessary
cover or tarpaulin, the ordinary splash of sea waves brought more water inside
the barge.
The petitioners wrote to Manila Bay a letter demanding payment for the loss and
another letter was sent to Pioneer claiming the amount under the insurance
policy but latter refused on the ground that its ability depended upon the Total
loss by Total loss of vessel only. Hence, petitioner commenced the civil case.
Petitioners: The implied warranty of seaworthiness provided for in the Insurance
Code refers only to the responsibility of the shipowner who must see to it that his
ship is reasonably fit to make in safety the contemplated voyage. A mere shipper
of cargo, having no control over the ship, has nothing to do with its
seaworthiness. A cargo owner has no control over the structure of the ship, its
cables, anchors, fuel and provisions, the manner of loading his cargo and the
cargo of other shippers, and the hiring of a sufficient number of competent
officers and seamen.
SC: Unmeritorious. There is no dispute over the liability of the common carrier
Manila Bay. In fact, it did not bother to appeal the questioned decision. However,
the petitioners state that Manila Bay has ceased operating as a firm and nothing
may be recovered from it. They are, therefore, trying to recover their losses from
the insurer.
The liability of the insurance company is governed by law. Section 113 of
the Insurance Code provides:
In every marine insurance upon a ship or freight, or freightage, or
upon anything which is the subject of marine insurance, a warranty is
implied that the ship is seaworthy.
Section 99 of the same Code also provides in part:
Marine insurance includes: (1) Insurance against loss of or damage
to: (a) Vessels, craft, aircraft, vehicles, goods, freights, cargoes,
merchandise x x x
The term "cargo" can be the subject of marine insurance and that once it
is so made, the implied warranty of seaworthiness immediately attaches
to whoever is insuring the cargo whether he be the shipowner or not. In
every contract of insurance upon anything which is the subject of marine

33
insurance, a warranty is implied that the ship shall be seaworthy at the time of
the inception of the voyage. Moreover, the fact that the unseaworthiness of
the ship was unknown to the insured is immaterial in ordinary marine
insurance and may not be used by him as a defense in order to recover
on the marine insurance policy.
Since the law provides for an implied warranty of seaworthiness in every contract
of ordinary marine insurance, it becomes the obligation of a cargo owner to look
for a reliable common carrier which keeps its vessels in seaworthy condition. The
shipper of cargo may have no control over the vessel but he has full
control in the choice of the common carrier that will transport his goods.
Or the cargo owner may enter into a contract of insurance which specifically
provides that the insurer answers not only for the perils of the sea but also
provides for coverage of perils of the ship.
We are constrained to apply Section 113 of the Insurance Code to the facts of this
case. In marine cases, the risks insured against are "perils of the sea." The
purpose of such insurance is protection against contingencies and against
possible damages and such a policy does not cover a loss or injury which must
inevitably take place in the ordinary course of things.

Perils of the sea:


extends only to losses caused by sea damage, or by the violence of the
elements, and does not embrace all losses happening at sea
losses from extraordinary occurrences only, such as stress of weather, winds
and waves, lightning, tempests, rocks and the like
include only such losses as are of extraordinary nature, or arise from some
overwhelming power, which cannot be guarded against by the ordinary
exertion of human skill and prudence
damage done to a vessel by perils of the sea includes every species of
damages done to a vessel at sea, as distinguished from the ordinary wear and
tear of the voyage, and distinct from injuries suffered by the vessel in
consequence of her not being seaworthy at the outset of her voyage (as in
this case)
everything which happens thru the inherent vice of the thing, or by the act of
the owners, master or shipper, shall NOT be reputed a peril, if not otherwise
borne in the policy
Petitioners: The loss of the cargo was caused by the perils of the sea, not by the
perils of the ship. As found by the trial court, the barge was turned loose from the
tugboat east of Cabuli Point "where it was buffeted by storm and waves."
Moreover, barratry (any willful misconduct on the part of the master or crew, in

Part 2_Case Digests


3.1_Insurance Law

34

pursuance of an unlawful or fraudulent purpose, without consent of the owner


and to the prejudice of the owners interest; still covered under perils of the sea)
against which the cargo was also insured, existed when the personnel of the
tugboat and the barge committed a mistake by turning loose the barge from the
tugboat east of Cabuli Point.

bad order condition. Choa Tiek Sieng filed a formal claim statement against the
vessel but the Filipino Merchants Insurance refused to pay the claim. Choa filed an
action with RTC.

SC: Unmeritorious. The facts clearly negate the petitioners' claim under the
insurance policy. The loss of the cargo was due to the perils of the ship rather
than the perils of the sea. The entrance of the sea water into the ship's hold
through the defective pipe was not due to any accident which happened during
the voyage, but to the failure of the ship's owner properly to repair a defect of the
existence of which he was apprised. The loss was therefore more analogous to
that which directly results from simple unseaworthiness than to that which result
from the perils of the sea.

Petitioners: An "all risks" marine policy has a technical meaning in insurance in


that before a claim can be compensable it is essential that there must be "some
fortuity, " "casualty" or "accidental cause" to which the alleged loss is attributable
and the failure of herein private respondent, upon whom lay the burden, to
adduce evidence showing that the alleged loss to the cargo in question was due
to a fortuitous event precludes his right to recover from the insurance policy.

Perils of the ship:


loss in the ordinary course of events
results from the natural and inevitable action of the sea, from the ordinary
wear and tear of the ship, or from the negligent failure of the ship's owner to
provide the vessel with proper equipment to convey the cargo under ordinary
conditions
GR: The insurer does not undertake to insure against perils of the ship.
X: To make insurer liable, there must some casualty something which could
not be foreseen as one of the necessary incidents of the adventure. The
purpose of the policy is to secure an indemnity against accidents which may
happen, not against events which must happen.
Therefore, the insurer is not liable. The shipowner excepts the perils of
the sea from his engagement under the bill of lading, while this is the
very perils against which the insurer intends to give protection. The
owners of the lost logs must look to the shipowner for redress and not
to the insurer.
61) FILIPINO MERCHANTS INSURANCE CO., INC VS COURT OF APPEALS
AND CHOA TIEK SENG,
G.R. NO. 85141, NOVEMBER 28, 1989
Facts: Choa Tiek Seng insured with Filipino Merchants Insurance the shipment of
600 metric tons (but actually was only 59.94 m. tons) of fishmeal in new gunny
bags of 90 kilos each against all risks under warehouse to warehouse terms. The
fishmeal were unloaded from the ship unto the arrastre contractor. The condition
of the bad order was reflected in the turn over survey report. The cargo was also
surveyed by the arrastre contractor before delivery of the cargo to the consignee
and the condition in such delivery was reflected covering a total of 227 bags in

RTC rendered decision in favor of Choa. The CA affirmed decision.

SC: Untenable. The "all risks clause" of the Institute Cargo Clauses read as
follows:
5. This insurance is against all risks of loss or damage to the subjectmatter insured but shall in no case be deemed to extend to cover loss,
damage, or expense proximately caused by delay or inherent vice or
nature of the subject-matter insured. Claims recoverable hereunder
shall be payable irrespective of percentage.
An "all risks policy" should be read literally as meaning all risks
whatsoever and covering all losses by an accidental cause of any kind.
The terms "accident" and "accidental", as used in insurance contracts, have
not acquired any technical meaning. They are construed by the courts in their
ordinary and common acceptance. Thus, the terms have been taken to mean that
which happens by chance or fortuitously, without intention and design,
and which is unexpected, unusual and unforeseen. An accident is an event
that takes place without one's foresight or expectation; an event that proceeds
from an unknown cause, or is an unusual effect of a known cause and, therefore,
not expected.
The very nature of the term "all risks" must be given a broad and
comprehensive meaning as covering any loss other than a willful and
fraudulent act of the insured. This is pursuant to the very purpose of an "all
risks" insurance to give protection to the insured in those cases where difficulties
of logical explanation or some mystery surround the loss or damage to property.
An "all asks" policy has been evolved to grant greater protection than that
afforded by the "perils clause," in order to assure that no loss can happen through
the incidence of a cause neither insured against nor creating liability in the ship; it
is written against all losses, that is, attributable to external causes.
Generally, the burden of proof is upon the insured to show that a loss arose from
a covered peril, but under an "all risks" policy the initial burden is on the

Part 2_Case Digests


3.1_Insurance Law

insured to prove that the cargo was in good condition when the policy attached
and that the cargo was lost, destroyed or deteriorated when unloaded from the
vessel; thereafter, the burden then shifts to the insurer to prove that the loss
was due to excepted perils.
There being no showing that the loss was caused by any of the excepted perils,
the insurer is liable under the policy. There is no evidence presented to show that
the condition of the gunny bags in which the fishmeal was packed was such that
they could not hold their contents in the course of the necessary transit, much
less any evidence that the bags of cargo had burst as the result of the weakness
of the bags themselves. Had there been such a showing that spillage would have
been a certainty, there may have been good reason to plead that there was no
risk covered by the policy. Under an 'all risks' policy, it was sufficient to
show that there was damage occasioned by some accidental cause of
any kind, and there is no necessity to point to any particular cause.
Filipino Merchants is to pay Choa P51,568.62 with interest at legal rate from the
date of the filing of the complaint.
62) CHOA TIEK SENG, doing business under the name and style of
SENG'S COMMERCIAL ENTERPRISES vs. HON. COURT OF APPEALS,
FILIPINO MERCHANTS' INSURANCE COMPANY, INC., BEN LINES
CONTAINER, LTD. AND E. RAZON, INC, G.R. No. 84507, [March 15, 1990] Deogracias
FACTS: On November 4, 1976 petitioner imported some lactose crystals from
Holland. The importation involved 600 6-ply paper bags with polyethylene inner
bags, each bag at 25 kilos net. The goods were loaded at the port at Rotterdam in
sea vans on board the vessel "MS Benalder as the mother vessel, and thereafter
aboard the feeder vessel "Wesser Broker V-25" of respondent Ben Lines
Container, Ltd. The goods were insured by the respondent Filipino Merchants'
Insurance Co., Inc. for P98,882.35, the equivalent of US$8,765.00 plus 50% markup against all risks under the terms of the insurance cargo policy. Upon
arrival at the port of Manila, the cargo was discharged into the custody of the
arrastre operator/broker respondent E. Razon, Inc., prior to the delivery to
petitioner. Of the 600 bags delivered to petitioner, 403 were in bad order. The
surveys showed that the bad order bags suffered spillage and loss later valued at
P33,117.63.
Petitioner filed a claim against respondent insurance company, which was
rejected, alleging that assuming that spillage took place while the goods were in
transit, petitioner and his agent failed to avert or minimize the loss by failing to
recover spillage from the sea van, thus violating the terms of the insurance policy
sued upon; and that assuming that the spillage did not occur while the cargo was
in transit, the said 400 bags were loaded in bad order, and that in any case, the

35
van did not carry any evidence of spillage. Choa filed complaint in RTC. RTC
dismissed case. CA affirmed dismissal.
CA: The cargo in question was insured in an "against all risk policy." Insurance
"against all risk" has a technical meaning in marine insurance. Under an "all risk"
marine policy, there must be a general rule be a fortuitous event in order to
impose liability on the insurer; losses occasioned by ordinary circumstances or
wear and tear are not covered, thus, while an "all risk" marine policy purports to
cover losses from casualties at sea, it does not cover losses occasioned by the
ordinary circumstances of a voyage, but only those resulting from extra and
fortuitous events.
ISSUE: Whether or not an "all risks" coverage covers only losses occasioned by or
resulting from "extra and fortuitous events" despite the clear and unequivocal
definition of the term made and contained in the policy sued upon.
HELD: No. CA erred. An all risk insurance policy insures against all causes of
conceivable loss or damage, except as otherwise excluded in the policy or due to
fraud or intentional misconduct on the part of the insured. It covers all losses
during the voyage whether arising from a marine peril or not, including pilferage
losses during the war.
In the present case, the "all risks" clause of the policy sued upon reads as follows:
This insurance is against all risks of loss or damage to the subject
matter insured but shall in no case be deemed to extend to cover
loss, damage, or expense proximately caused by delay or inherent
vice or nature of the subject matter insured. Claims recoverable
hereunder shall be payable irrespective of percentage.
The insurance policy covers all loss or damage to the cargo except those caused
by delay or inherent vice or nature of the cargo insured. It is the duty of the
respondent insurance company to establish that said loss or damage falls within
the exceptions provided for by law, otherwise it is liable therefor. In this case, the
damage caused to the cargo has not been attributed to any of the
exceptions provided for nor is there any pretension to this effect. Thus,
respondent insurance company must pay.
63) DELSAN TRANSPORT LINES, INC. vs. THE HON. COURT OF APPEALS
and AMERICAN HOME ASSURANCE CORPORATION, G.R. No. 127897,
[November 15, 2001]
FACTS: Caltex Philippines entered into a contract of affreightment with the
Delsan Transport Lines, Inc., whereby the said common carrier agreed to transport
Caltex's industrial fuel oil from the Batangas-Bataan Refinery to different parts of
the country. The shipment was insured with American Home Assurance

Part 2_Case Digests


3.1_Insurance Law

Corporation. On August 14, 1986, petitioner's vessel, the MT Maysun, set sail from
Batangas for Zamboanga City. Unfortunately, the vessel sank taking with it the
entire cargo of fuel oil. Private respondent paid Caltex the sum of P5,096,635.57
representing the insured value of the lost cargo. Exercising its right of subrogation
under Article 2207 of the New Civil Code, the private respondent demanded of
the petitioner the same amount it paid to Caltex.
Due to its failure to collect from the petitioner despite prior demand, private
respondent filed a complaint with the RTC for collection of a sum of money. RTC
dismissed complaint, finding that the vessel, MT Maysun, was seaworthy to
undertake the voyage as determined by the Philippine Coast Guard per Survey
Certificate Report upon inspection during its annual dry-docking and that the
incident was caused by unexpected inclement weather condition or force
majeure, thus exempting petitioner from liability for the loss of its cargo.
On appeal, CA reversed decision.
Delsan: When American Homes paid Caltex, it was equivalent to a tacit
recognition that the ill-fated vessel was seaworthy; otherwise, the former was not
legally liable to latter due to the latter's breach of implied warranty under the
marine insurance policy that the vessel was seaworthy.
ISSUES:
1. WON the payment made by American Home to Caltex for the insured
value of the lost cargo amounted to an admission that the vessel was
seaworthy, thus precluding any action for recovery against the petitioner.
2. WON the non-presentation of the marine insurance policy bars the
complaint for recovery of sum of money for lack of cause of action.
HELD:
1. NO. The payment made by American Home for the insured value of the
lost cargo operates as waiver of its right to enforce the term of the implied
warranty against Caltex under the marine insurance policy.
However, the same cannot be validly interpreted as an automatic
admission of the vessels seaworthiness by American Home as to
foreclose recourse against Delsan for any liability under its contractual
obligation as a common carrier. The fact of payment grants American Home
subrogatory right which enables it to exercise legal remedies that would
otherwise be available to Caltex as owner of the lost cargo against Delsan, the
common carrier.

36
From the nature of their business and for reasons of public policy, common
carriers are bound to observe extraordinary diligence in the vigilance over
the goods and for the safety of passengers transported by them, according to all
the circumstances of each case. In the event of loss, destruction or
deterioration of the insured goods, common carriers shall be responsible
unless the same is brought about, among others, by flood, storm,
earthquake, lightning or other natural disaster or calamity. In all other
cases, if the goods are lost, destroyed or deteriorated, common carriers are
presumed to have been at fault or to have acted negligently, unless they prove
they observed extraordinary diligence.
In order to escape liability for the loss of its cargo of industrial fuel oil belonging to
Caltex, Delsan attributes the sinking of MT Maysun to fortuitous event or force
majeure. But the testimony of the captain and chief mate that there were strong
winds and waves 20 feet high was effectively rebutted and belied by the weather
report of PAGASA. Thus, as the CA correctly ruled, Delsans vessel, MT Maysun,
sank with its entire cargo for the reason that it was not seaworthy. There was no
squall or bad weather or extremely poor sea condition in the vicinity where the
said vessel sank.
Additionally, the exoneration of MT Maysuns officers and crew merely concern
their respective administrative liabilities. It does not in any way operate to
absolve Delsan the common carrier from its civil liability arising from its failure to
observe extraordinary diligence in the vigilance over the goods it was
transporting and for the negligent acts or omissions of its employees, the
determination of which properly belongs to the courts. In the case at bar, Delsan
is liable for the insured value of the lost cargo of industrial fuel oil
belonging to Caltex for its failure to rebut the presumption of fault or
negligence as common carrier occasioned by the unexplained sinking of
its vessel, MT Maysun, while in transit.

2. NO. The presentation in evidence of the marine insurance policy is not


indispensable in this case before the insurer may recover from the
common carrier the insured value of the lost cargo in the exercise of its
subrogatory right. The subrogation receipt, by itself, is sufficient to
establish not only the relationship of American Home as insurer
and Caltex, as the assured shipper of the lost cargo of industrial
fuel oil, but also the amount paid to settle the insurance claim.
The right of subrogation accrues simply upon payment by the insurance
company of the insurance claim.

Part 2_Case Digests


3.1_Insurance Law

64) FRANCISCO DEL VAL, ET AL. vs. ANDRES DEL VAL,, G.R. No. 9374,
[February 16, 1915]
FACTS: The parties are siblings who were the only heirs at law and next of kin of
Gregorio del Val, who passed away intestate. During the lifetime of the deceased
he took out insurance on his life for the sum of P40,000 and made it payable to
Andres del Val as sole beneficiary. After his death, the defendant Andres collected
the face of the policy. He paid the sum of P18,365.20 to redeem certain real
estate which the decedent had sold to third persons with a right to repurchase.
The redemption of said premises was made by the attorney of the defendant in
the name of the petitioners and Andres as heirs of the deceased vendor. Andres,
on death of the Gregorio, took possession of most of the latters personal property
and also the balance on the insurance policy amounting to P21,634.80.
Plaintiffs: The amount of the insurance policy belonged to the estate of the
deceased and not to the defendant personally, hence they are entitled to a
partition not only of the real and personal property, but also of the P40,000 life
insurance. The complaint prays a partition of all the property, both real and
personal, left by the deceased, and that the defendant account for P21,634.80.
They also wanted to divide this equally among the plaintiffs and defendant along
with the other property of deceased.
Defendant Andres: Redemption of the real estate sold by his father was made
in the name of the plaintiffs and himself instead of in his name alone without his
knowledge or consent. He also averred that it was not his intention to use the
proceeds of the insurance policy for the benefit of any person but himself, he
alleging that he was and is the sole owner thereof and that it is his individual
property.
ISSUE: Can the proceeds of the policy be divided among the heirs?
HELD: NO. The proceeds of the life-insurance policy belong exclusively to the
defendant as his individual and separate property. The proceeds of an insurance
policy belong exclusively to the beneficiary and not to the estate of the person
whose life was insured, and such proceeds are the separate and individual
property of the beneficiary, and not of the heirs of the person whose life was
insured. This doctrine in America is embedded in the Code of Commerce where:
The amount which the underwriter must deliver to the person
insured, in fulfillment of the contract, shall be the property of the
latter, even against the claims of the legitimate heirs or creditors of
any kind whatsoever of the person who effected the insurance in
favor of the former.
The contract of life insurance is a special contract and the destination of the
proceeds is determined by special laws which deal exclusively with that subject.

37
The Civil Code has no provisions which relate directly and specifically to lifeinsurance contracts or to the destination of life insurance proceeds. That was
under the Code of Commerce.
The plaintiffs claim that the property repurchased with the insurance proceeds
belongs to the heirs in common and not to the defendant alone. This wasnt
agreed upon by the court unless the facts appeared that Andres acted as he did
with the intention that the other heirs should enjoy with him the ownership of the
estate.

65) THE BANK OF THE PHILIPPINE ISLANDS, administrator of the estate


of the late Adolphe Oscar Schuetze, vs. JUAN POSADAS, JR., Collector of
Internal Revenue, Jr., G.R. No. 34583, [1931]
FACTS: BPI, as administrator, appealed to CFI absolving defendant, Collector of
Internal Revenue, from the complaint filed against it in recovering the inheritance
tax amounting to P1,209 paid by Rosario Gelano Vda de Schuetze, under protest,
and sum of P20,150 representing the proceeds of the insurance policy of the
deceased.
Rosario and Adolphe were married in January 1914. The wife was actually
residing and living in Germany when Adolphe died in December 1927. The latter
while in Germany, executed a will in March 1926, pursuant with its law wherein
Rosario was named his universal heir. The deceased possessed not only real
property situated in the Philippines but also personal property consisting of shares
of stocks in 19 domestic corporations. Included in the personal property is a
life insurance policy issued at Manila on January 1913 for the sum of
$10,000 by the Sun Life Assurance Company of Canada, Manila Branch.
In the insurance policy, the estate of the deceased was named the beneficiary
without any qualification. Rosario is the sole and only heir of the deceased. BPI,
as administrator of the decedents estate and attorney in fact of Rosario, having
been demanded by Posadas to pay the inheritance tax, paid under protest.
Notwithstanding various demands made by plaintiff, Posadas refused to refund
such amount.
ISSUE: Whether the proceeds of the life insurance policy is paraphernal or
community property. (if community, then not subject to inheritance tax)
HELD: As all the premiums on the life-insurance policy taken out by the late
Adolphe
Oscar
Schuetze,
were
paid
out of the conjugal
funds,
with the exception of the first,
the proceeds of the policy,
excluding the proportional part corresponding to the first premium, constitute
community property, notwithstanding the fact that the policy was made
payable to the deceased's estate, so that one-half of said proceeds belongs

Part 2_Case Digests


3.1_Insurance Law

38

to the estate, and the other half to the deceased's widow, Rosario. CIR is
ordered to return to BPI the one-half of the tax collected upon the amount of
P20,150, being the proceeds of the insurance policy on the life of the late
Adolphe, after deducting the proportional part corresponding to the first premium.

of P5,882.00 plus the additional benefits for accidental death also in the amount
of P5,882.00 and the refund of P18.00 paid for the premium due November, 1969,
minus the unpaid premiums and interest thereon due for January and February,
1969, in the sum of P36.27.

Both according to our Civil Code and to the ruling of those North American States
where the Spanish Civil Code once governed, the proceeds of a life-insurance
policy whereon the premiums were paid with conjugal money, belong to
the conjugal partnership.

Carponia filed with the insurer a claim for the proceeds of the policy as the
designated beneficiary therein, although she admits that she and the insured
Buenaventura were merely living as husband and wife without the benefit of
marriage. Pascuala Vda. de Ebrado also filed her claim as the widow of the
deceased insured. She asserts that she is the one entitled to the insurance
proceeds, not the common-law wife, Carponia.

A
life-insurance
policy
belongs
exclusively
to the beneficiary
upon the death of the person insured, and that in the present case, as Adolphe
named his own estate as the sole beneficiary of the insurance on his life,
upon his death the latter became the sole owner of the proceeds, which
therefore became subject to the inheritance tax. An heir appointed
beneficiary to a life- insurance policy taken out by the deceased,
becomes the absolute owner of the proceeds of such policy upon the death of the
insured.
The proceeds of a life-insurance policy payable to the insured's estate, on which
the premiums were paid by the conjugal partnership, constitute community
property, and belong one-half to the husband and the other half to the wife,
exclusively. But if the premiums were paid partly with paraphernal and partly
conjugal funds, the proceeds are likewise in like proportion paraphernal in part
and conjugal in part.
The proceeds of a life-insurance policy payable to the insured's estate as the
beneficiary, if delivered to the testamentary administrator of the former as part of
the assets of said estate under probate administration, are subject to the
inheritance tax according to the law on the matter, if they belong to the assured
exclusively, and it is immaterial that the insured was domiciled in these Islands or
outside.
66) THE INSULAR LIFE ASSURANCE COMPANY, LTD. vs. CARPONIA T.
EBRADO and PASCUALA VDA. DE EBRADO, G.R. No. L-44059, [October 28,
1977]
FACTS: On September 1, 1968, Buenaventura Cristor Ebrado was issued by the
Insular Life Assurance Co., Ltd., a policy on a whole-life plan for P5,882 with a
rider for Accidental Death Benefits for the same amount. He designated Carponia
T. Ebrado as the revocable beneficiary in his policy. He referred to her as his wife.
On October 21, 1969, Buenventura C. Ebrado died as a result of an accident when
he was hit by a falling branch of a tree. As the insurance policy was in force, The
Insular Life Assurance Co., Ltd. stands liable to pay the coverage of the policy in
an amount of P11,745.73, representing the face value of the policy in the amount

ISSUE: Can the common-law wife, Carponia Ebrado, named as beneficiary in


the life insurance policy of a legally married man claim the proceeds thereof in
case of death of the latter?
HELD: NO. The general rules of civil law should be applied to resolve matters not
specifically provided in the Insurance Law. Article 2011 of the New Civil Code
states:
The contract of insurance is governed by special laws. Matters not
expressly provided for in such special laws shall be regulated by this
Code.
And under Article 2012 of the same Code:
Any person who is forbidden from receiving any donation under
Article 739 cannot be named beneficiary of a fife insurance policy by
the person who cannot make a donation to him.
Common-law spouses are, definitely, barred from receiving donations from each
other. Also, conviction for adultery or concubinage is not required as only
preponderance of evidence is necessary.
In essence, a life insurance policy is no different from a civil donation insofar as
the beneficiary is concerned. Both are founded upon the same consideration:
liberality. A beneficiary is like a donee, because the premiums of the policy which
the insured pays out of liberality, the beneficiary will receive the proceeds or
profits of said insurance.

Part 2_Case Digests


3.1_Insurance Law

M. Cash Surrender Value - Herrera


(GR No. L-2910, 1951)
67) THE MANUFACTURERS LIFE INSURANCE CO., plaintiff-appellant, vs.
BIBIANO L. MEER, in the capacity as Collector of Internal
Revenue, defendant-appellee
Facts: The Manufacturers Life Insurance Company is duly registered and licensed
to engage in life insurance business in the Philippines for more than five years
until 1941. But due to the exigencies of the war it closed the branch office at
Manila during 1942 up to September 1945. In the course of its operations before
the war, plaintiff issued a number of life insurance policies in the Philippines
containing nonforfeiture clauses, viz:
8. Automatic Premium Loan This Policy shall not lapse for nonpayment of any premium after it has been three full years in
force, if, at the due date of such premium, the Cash Value of this
Policy and of any bonus additions and dividends left on
accumulation (after deducting any indebtedness to the Company
and the interest accrued thereon) shall exceed the amount of said
premium. In which event the company will, without further
request, treat the premium then due as paid, and the amount of
such premium, with interest from its actual due date at six per
cent per annum, compounded yearly, and one per cent,
compounded yearly, for expenses, shall be a first lien on this
Policy in the Company's favour in priority to the claim of any
assignee or any other person. The accumulated lien may at any
time, while the Policy is in force, be paid in whole or in part xxx
From January 1, 1942 to December 31, 1946 for failure of the insured under the
above policies to pay the corresponding premiums for one or more years, the
plaintiff's head office at Toronto, applied the provisions of the automatic
premium loan clauses; and the net amount of premiums so advanced or loaned
totalled P1,069,254.98. On this sum the defendant CIR assessed P17,917.12
which plaintiff paid supra protest. The assessment was made pursuant to section
255 of the National Internal Revenue Code as amended, which partly provides:
SEC. 255. Taxes on insurance premiums There shall be
collected from every person, company, or corporation (except
purely cooperative companies or associations) doing insurance
business of any sort in the Philippines a tax of one per centum of
the total premiums collected . . . whether such premiums are paid
in money, notes, credits, or any substitute for money but
premiums refunded within six months after payment on account
of rejection of risk or returned for other reason to person insured
shall not be included in the taxable receipts . . . .

39
Plantiff: When there are premium loans or premium advances, as above stated,
by virtue of the non-forfeiture clauses, it did not collect premiums within the
meaning of the above sections of the law, and therefore it is not amenable to the
tax therein provided.

Issues:
1. WON premium advances made by plaintiff-appellant under the automatic
premium loan clause of its policies are 'premiums collected' by the Company
subject to tax.
2. WON in the application of the automatic premium loan clause of plaintiffappellant's policies, there is payment in money, notes, credits, or any
substitutes for money.
3. WON the collection of the alleged deficiency premium taxes constitutes double
taxation.
4. Whether the making of premium advances, granting for the sake of argument
that it amounted to collection of premiums, were done in Toronto, Canada, or in
the Philippines.
5. WON the fact that plaintiff-appellant was not doing business in the Philippines
from January 1, 1942 to September 30, 1945 exempts it from payment of
premium taxes corresponding to said period.
Held:
Illustration: Suppose that 'A', 30 years of age, secures a 20-year endowment
policy for P5,000 from plaintiff-appellant Company and pays an annual premium
of P250. 'A' pays the first ten yearly premiums amounting to P2,500 and on this
amount plaintiff-appellant pays the corresponding taxes under section 255 of the
NIRC. Suppose also that the cash value of said policy after the payment of the
10th annual premium amounts to P1,000. When on the 11th year the annual
premium fell due and the insured remitted no money within the month's grace,
the insurer treated the premium then over due as paid from the cash value, the
amount being a loan to the policyholder who could discharge it at any time with
interest at 6 per cent. The insurance contract, therefore, continued in force for the
11th year.
1. Yes. "How could there be such a collection" plaintiff argues "when as a result
thereof, insurer becomes a creditor, acquires a lien on the policy and is entitled
to collect interest on the amount of the unpaid premiums?" Wittingly or
unwittingly, the "premium" and the "loan" have been interchanged in the
argument. The insurer "became a creditor" of the loan, but not of the premium

Part 2_Case Digests


3.1_Insurance Law

that had already been paid. And it is entitled to collect interest on the loan, not
on the premium. In other words, "A" paid the premium for the eleventh year;
but in turn he became a debtor of the company for the sum of P250. This debt
he could repay either by later remitting the money to the insurer or by letting
the cash value compensate for it. The debt may also be deducted from the
amount of the policy should "A" die thereafter during the continuance of the
policy. The debt may also be deducted from the amount of the policy should
the insured die thereafter during the continuance of the policy.
There was an increase in the assets of the insurer. There was the new credit for
the advances made. True, the plaintiff could not sue the insured to enforce that
credit. But it has means of satisfaction out of the cash surrender value. If the
credit is paid out of the cash surrender value, there were no new funds added
to the company's assets. Cash surrender value "as applied to a life
insurance policy, is the amount of money the company agrees to pay to
the holder of the policy if he surrenders it and releases his claims
upon it. The more premiums the insured has paid the greater will be the
surrender value; but the surrender value is always a lesser sum than the total
amount of premiums paid."
The cash value or cash surrender value is therefore an amount which
the insurance company holds in trust for the insured to be delivered
to him upon demand. It is therefore a liability of the company to the
insured. Now then, when the company's credit for advances is paid out of the
cash value or cash surrender value, that value and the company's liability is
thereby diminished pro tanto. The decrease of a person's liabilities means a
corresponding increase in his net assets.
2. Yes. The insurer agreed to consider the premium paid on the strength of
the automatic loan. The premium was therefore paid by means of a "note"
or "credit" or "other substitute for money" and the tax is due because
section 255 above quoted levies taxes according to the total premiums
collected by the insurer "whether such premiums are paid in
money, notes, credits or any substitute for money.

3. No. Appellant goes back to the illustration, "A failed to pay the premium on
the 11th year and the insurer advanced P250 from the cash value. If the
amount of P250 is deducted from the cash value of P1,000 of the policy,
then taxing this P250 anew as premium collected, as was done in the
present case, will amount to double taxation since taxes had already been
collected on the cash value of P1,000 as part of the P2,500 collected as
premiums for the first ten years." The trouble with the argument is that it
assumes all advances are necessarily repaid from the cash value. That is

40
true in some cases. In others the insured subsequently remits the
money to repay the advance and to keep unimpaired the cash reserve of
his policy. Of the total amount advanced (P1,069,255) P158,667 had
actually been repaid at the time of assessment notice. Besides, the
premiums paid and on which taxes had already been collected, were those
for the 10 years. The tax demanded is on the premium for the 11th year.
Further, there is no constitutional prohibition against double taxation.

4. Philippines. Appellant: as the advances of premiums were made in Toronto,


such premiums are deemed to have been paid there not in the
Philippines and therefore those payments are not subject to local
taxation. The law does not contemplate premiums collected in the
Philippines. Subscribing to this would make foreign insurers evade the tax
by contriving to require that premium payments shall be made at their
head offices. It is enough that the insurer is doing insurance business in
the Philippines, irrespective of the place of its organization or
establishment. In any event there is no constitutional prohibition against
double taxation.

5. Untenable. Although during those years the appellant was not open for
new business because its branch office was closed, still it was practically
and legally, operating in this country by collecting premiums on its
outstanding policies, incurring the risks and/or enjoying the benefits
consequent thereto, without having previously taken any steps indicating
withdrawal in good faith from this field of economic activity. Further, in
objecting to the payment of the tax, plaintiff-appellant never insisted,
before the BIR that it was not engaged in business in this country during
those years.

N. Suretyship
GR No. 109937, 1994
68) DEVELOPMENT BANK OF THE PHILIPPINES, petitioner, vs. COURT OF
APPEALS and the ESTATE OF THE LATE JUAN B. DANS, represented by
CANDIDA G. DANS, and the DBP MORTGAGE REDEMPTION INSURANCE
POOL, respondents
Facts:
In May 1987, Juan B. Dans, together with his family, applied for a loan of
P500,000 with DBP Basilan Branch. As the principal mortgagor, Dans, then 76
years of age, was advised by DBP to obtain a mortgage redemption insurance

Part 2_Case Digests


3.1_Insurance Law

with the DBP Mortgage Redemption Insurance Pool (DBP MRI Pool). A loan, in the
reduced amount of P300,000, was approved and released by DBP. From the
proceeds of the loan, DBP deducted the amount of P1,476 as payment for the MRI
premium. Dans accomplished and submitted the "MRI Application for Insurance"
and the "Health Statement for DBP MRI Pool." The MRI premium of Dans, less the
DBP service fee of 10 percent, was credited by DBP to the savings account of the
DBP MRI Pool. Accordingly, the DBP MRI Pool was advised of the credit.
On September 3, 1987, Dans died of cardiac arrest. The DBP, upon notice, relayed
this information to the DBP MRI Pool. On September 23, 1987, the DBP MRI Pool
notified DBP that Dans was not eligible for MRI coverage, being over the
acceptance age limit of 60 years at the time of application. DBP apprised Candida
Dans of the disapproval of her late husband's MRI application. The DBP offered to
refund the premium of P1,476, but Candida refused to accept it, demanding
payment of the face value of the MRI or an amount equivalent to the loan. She,
likewise, refused to accept anex gratia settlement of P30,000, which the DBP later
offered.
Respondent Estate, through Candida Dans as administratrix, filed a complaint
with RTC against DBP and the insurance pool for "Collection of Sum of Money with
Damages." RTC decided in favor of respondent Estate and against DBP. The DBP
MRI Pool, however, was absolved from liability, after the trial court found no
privity of contract between it and the deceased. The trial court declared DBP in
estoppel for having led Dans into applying for MRI and actually collecting the
premium and the service fee, despite knowledge of his age ineligibility. The CA
affirmed in toto.
Issue: WON DBP and DBP MRI Pool are liable.
Held: DBP, yes and DBP MRI Pool, no. As to DBP MRI Pool, it did not approve the
application of Dans. There is also no showing that it accepted the sum of P1,476,
which DBP credited to its account with full knowledge that it was payment for
Dan's premium. Under the provisions of the Health Statement for DBP Pool, the
MRI coverage shall take effect: (1) when the application shall be approved by the
insurance pool; and (2) when the full premium is paid during the continued good
health of the applicant. These two conditions, being joined conjunctively, must
concur. There was no perfected contract of insurance; hence, the DBP MRI Pool
cannot be held liable on a contract that does not exist.
As to DBP, it was wearing two legal hats: the first as a lender, and the second as
an insurance agent.
In dealing with Dans, DBP As an insurance agent, DBP made Dans go through the
motion of applying for said insurance, thereby leading him and his family to
believe that they had already fulfilled all the requirements for the MRI and that
the issuance of their policy was forthcoming. Apparently, DBP had full knowledge

41
that Dan's application was never going to be approved. The maximum age for MRI
acceptance is 60 years as clearly and specifically provided in the Group Mortgage
Redemption Insurance Policy. Under Article 1987 of the Civil Code of the
Philippines, "the agent who acts as such is not personally liable to the party with
whom he contracts, unless he expressly binds himself or exceeds the limits of his
authority without giving such party sufficient notice of his powers." The DBP is not
authorized to accept applications for MRI when its clients are more than 60 years
of age. Knowing all the while that Dans was ineligible for MRI coverage because of
his advanced age, DBP exceeded the scope of its authority when it accepted
Dan's application for MRI by collecting the insurance premium, and deducting its
agent's commission and service fee. There is no showing that Dans knew of the
limitation on DBP's authority to solicit applications for MRI. If the third person
dealing with an agent is unaware of the limits of the authority conferred by the
principal on the agent and he (third person) has been deceived by the nondisclosure thereof by the agent, then the latter is liable for damages to him.
Inasmuch as the non-disclosure of the limits of the agency carries with it the
implication that a deception was perpetrated on the unsuspecting client, the
provisions of Articles 19, 20 and 21 of the Civil Code of the Philippines come into
play.
Article 19 Every person must, in the exercise of his rights and in the
performance of his duties, act with justice give everyone his due and
observe honesty and good faith.
Article 20 Every person who, contrary to law, willfully or negligently
causes damage to another, shall indemnify the latter for the same.
Article 21 Any person, who willfully causes loss or injury to another
in a manner that is contrary to morals, good customs or public policy
shall compensate the latter for the damage.
The DBP's liability, however, cannot be for the entire value of the insurance policy.
To assume that were it not for DBP's concealment of the limits of its authority,
Dans would have secured an MRI from another insurance company, and therefore
would have been fully insured by the time he died, is highly speculative.
Considering his advanced age, there is no absolute certainty that Dans could
obtain an insurance coverage from another company. It must also be noted that
Dans died almost immediately, i.e., on the nineteenth day after applying for the
MRI, and on the twenty-third day from the date of release of his loan. One is
entitled to an adequate compensation only for such pecuniary loss suffered by
him as he has duly proved. Damages, to be recoverable, must not only be
capable of proof, but must be actually proved with a reasonable degree of
certainty. Speculative damages are too remote to be included in an accurate
estimate of damages.

Part 2_Case Digests


3.1_Insurance Law

WHEREFORE, the decision of the Court of Appeals is MODIFIED and petitioner DBP
is ORDERED: (1) to REIMBURSE respondent Estate of Juan B. Dans the amount of
P1,476 with legal interest from the date of the filing of the complaint until fully
paid; and (2) to PAY said Estate P50,000 as moral damages and P10,000 as
attorney's fees. With costs against petitioner.
Note: Wa bitaw ko kasabot ngano ni-fall under Suretyship ni siya nga topic, kay
wa jud na namention maski kausa. Basin ang pgconstitute sa Mortgage
Redemption Insurance Contract? Which in this case was not perfected.

42

Part 2_Case Digests


3.1_Insurance Law

43

O. Claims Settlement - Herrera


GR No. 76101-02, 1991
69) TIO KHE CHIO, petitioner, vs. THE HONORABLE COURT OF APPEALS and
EASTERN ASSURANCE AND SURETY CORPORATION, respondents
Facts: On December 18, 1978, Tio Khe Chio imported 1,000 bags of fishmeal
valued at $36,000.30 from Agro Impex, Texas, USA. The goods were insured with
respondent EASCO and shipped on board the M/V Peskov, a vessel owned by Far
Eastern Shipping Company. When the goods reached Manila, they were found to
have been damaged by sea water which rendered the fishmeal useless. Petitioner
filed a claim with EASCO and Far Eastern Shipping. Both refused to pay. Petitioner
sued them before the then CFI for damages. EASCO, as the insurer, filed a
counterclaim against the petitioner for the recovery of P18,387.86 representing
the unpaid insurance premiums.
CFI ordered EASCO and Far Eastern Shipping to pay petitioner solidarily the sum
of P105,986.68 less the amount of P18,387.86 for unpaid premiums with interest
at the legal rate from the filing of the complaint, the sum of P15,000.00 as
attorney's fees and the costs. The judgment became final as to EASCO but the
shipping company appealed to the CA and was absolved from liability by the said
court.
The trial court, upon motion by petitioner, issued a writ of execution against
EASCO. The sheriff enforcing the writ reportedly fixed the legal rate of interest at
12%. Respondent EASCO moved to quash the writ alleging that the legal interest
to be computed should be 6%. The trial court denied EASCO's motion. EASCO
then filed a petition for certiorari and prohibition before the Court of Appeals.
CA held that the interest that the private respondent is entitled to collect from the
petitioner is hereby reduced to 6% per annum.
Issue: What is the legal rate of interest to be imposed in actions for damages
arising from unpaid insurance claims? Petitioner Tio Khe Chio claims that it should
be 12% pursuant to Articles 243 and 244 of the Insurance Code while EASCO
claims that it should be 6% under Article 2209 of the Civil Code.
Held: 6%.
Section 243 of the Insurance Code provides:
xxx Refusal or failure to pay the loss or damage within the
time prescribed herein will entitle the assured to collect interest
on the proceeds of the policy for the duration of the delay at the
rate of twice the ceiling prescribed by the Monetary
Board, unless such failure or refusal to pay is based on the
ground that the claim is fraudulent.

Section 244 of the aforementioned Code also provides:


In case of any litigation for the enforcement of any policy or
contract of insurance, it shall be the duty of the Commissioner or
the Court, as the case may be, to make a finding as to whether
the payment of the claim of the insured has been unreasonably
denied or withheld; and in the affirmative case, the insurance
company shall be adjudged to pay damages which shall consist of
attorney's fees and other expenses incurred by the insured
person by reason of such undeniable denial or withholding of
payment plus interest of twice the ceiling prescribed by the
Monetary Board of the amount of the claim due the insured xxx
The CA made no finding that there was an unjustified refusal or withholding of
payment on petitioner's claim. In fact, the CA had this to say:
xxx EASCO's refusal to settle the claim to Tio Khe Chio was based
on some ground which, while not sufficient to free it from liability
under its policy, nevertheless is sufficient to negate any assertion
that in refusing to pay, it acted unjustifiably. xxx The case posed
some genuine issues of interpretation of the terms of the policy
as to which persons may honestly differ. This is the reason the
trial court did not say EASCO's refusal was unjustified.
So the aforecited sections of the Insurance Code are not pertinent to the instant
case. They apply only when the court finds an unreasonable delay or refusal in
the payment of the claims.
Neither does Circular No. 416 of the Central Bank which took effect on July 29,
1974 pursuant to PD 116 (Usury Law) which raised the legal rate of interest from
6% to 12% apply to the case at bar as contended by the petitioner. The adjusted
rate mentioned in the circular refers only to loans or forbearances of money,
goods or credits and court judgments thereon but not to court judgments for
damages arising from injury to persons and loss of property which does not
involve a loan.
The legal rate of interest is 6% per annum, and not 12%, where a
judgment award is based on an action for damages for personal injury,
not use or forbearance of money, goods or credit. The interest by way of
damages is governed by Article 2209 of the Civil Code, which reads:

Part 2_Case Digests


3.1_Insurance Law

If the obligation consists in the payment of a sum of money and


the debtor incurs in delay, the indemnity for damages, there
being no stipulation to the contrary, shall be the payment of
interest agreed upon, and in the absence of stipulation, the legal
interest which is six per cent per annum.
Since the contending parties did not allege the rate of interest stipulated in the
insurance contract, the legal interest was properly pegged by the Appellate Court
at 6%.

44

Part 2_Case Digests


3.1_Insurance Law

GR No. 138737, 2001


70) FINMAN GENERAL ASSURANCE CORPORATION, petitioner, vs. COURT
OF APPEALS and USIPHIL INCORPORATED, respondents
Facts: Usiphil Inc. insured certain properties (office, furniture, fixtures, shop
machinery and other trade equipment) from fire with Finman (then doing business
under the name Summa Insurance Corporation). Sometime in 1982, Usiphil filed
an insurance claim amounting to P987,126.11 for the loss of the insured
properties due to fire. Finman appointed Adjuster H.H. Bayne to undertake the
valuation and adjustment of the loss. Usiphil submitted its Sworn Statement of
Loss and Formal Claim, signed by Reynaldo Cayetano, Usiphils Manager, and a
Proof of Loss signed by its Accounting Manager Pedro Palallos and countersigned
by H.H. Bayne's Adjuster F.C. Medina.
Palallos personally followed-up private respondent's claim with Finmans President
Joaquin Ortega. During their meeting, Ortega instructed their Finance Manager,
Rosauro Maghirang, to reconcile the records. Thereafter, Maghirang and Palallos
signed a Statement/Agreement, dated February 28, 1985, which indicated that
the amount due respondent was P842,683.40.
Despite repeated demands, petitioner refused to pay the insurance claim. Thus,
private respondent was constrained to file a complaint against petitioner for the
unpaid insurance claim. Finman maintained that Usiphil cannot recover because it
failed to comply with Policy Condition No. 13 regarding the submission of certain
documents to prove the loss. RTC ruled in favor of Usiphil. CA modified decision as
to #1 below, but affirmed in toto the others: Finman is
1. To pay the plaintiff the sum of P842,683.40 and to pay 24%
interest per annum from 03 May 1985 until fully paid;
2. To pay the plaintiff the sum equivalent to 10% of the principal
obligation as and for attorney's fees, plus P1,500.00 per court
appearance of counsel;
3. To pay the plaintiff the amount of P30,000.00 as exemplary
damages in addition to the actual and compensatory damages
awarded xxx
Issues:
1. Was there compliance by Usiphil of Condition No. 13?
2. Was the 24% interest rate proper?
Held:

45
1. Yes. Both the trial court and the CA concur in holding that private respondent
had substantially complied with Policy Condition No. 13. Requirements under
which were allegedly communicated to private respondent in the two letters
of H.H. Bayne to private respondent. The letter stated, among others:
To be able to expedite adjustment of this case, please submit to
us without delay the following documents and/or particulars:
For FFF, Machineries/Equipment Claims and For Stock Claim
1. Your formal claim (which may be accomplished in the enclosed
form) accompanied by a detailed inventory of the documents
submitted.
2. Certification from the appropriate government office indicating
the date of the occurrence of the fire, the property involved, its
location and possible point of origin.
3. Proof of premium payment.
4. Three
color
photographs
of
the
debris
properly
captioned/identified/dated and initiated by the claimant at the
back.
Though our adjusters will also take photographs in the manner
prescribed above, please do not rely on his photographs in the
preservations of your evidence of loss thru pictures.
5. Copies of purchase invoices xxx
Factual findings and conclusions of the trial court and the CA are entitled to great
weight and respect, and will not be disturbed on appeal in the absence of any
clear showing that the trial court overlooked certain facts or circumstances which
would substantially affect the disposition of the case. There is no cogent reason to
deviate from this salutary rule in the present case.
Policy Condition No. 13 reads:
The insured shall give immediate written notice to the Company
of any loss, protect the property from further damage, forthwith
separate the damaged and undamaged personal property, put it
in the best possible order, furnish a complete inventory of the
destroyed, damaged, and undamaged property, showing in detail
quantities, costs, actual cash value and the amount of loss
claimed; AND WITHIN SIXTY DAYS AFTER THE LOSS, UNLESS SUCH
TIME IS EXTENDED IN WRITING BY THE COMPANY, THE INSURED
SHALL RENDER TO THE COMPANY A PROOF OF LOSS, signed
and sworn to by the insured, stating the knowledge and belief of

Part 2_Case Digests


3.1_Insurance Law

the insured as to the following: the time and origin of the loss, the
interest of the insured and of all others in the property, the actual
cash value of each item thereof and the amount of loss thereto,
all encumbrances thereon, all other contracts of insurance,
whether valid or not, covering any of said property, any changes
in the title, use, occupation, location, possession or exposures of
said property since the issuing of this policy by whom and for
what purpose any buildings herein described and the several
parts thereof were occupied at the time of loss and whether or
not it then stood on leased ground, and shall furnish a copy of all
the descriptions and schedules in all policies, and if required
verified plans and specifications of any building, fixtures, or
machinery destroyed or damaged. The insured, as often as may
be reasonably required, shall exhibit to any person designated by
the company all that remains of any property herein described,
and submit to examination under oath by any person named by
the Company, and subscribe the same; and, as often as may be
reasonably required, shall produce for examination all books of
account, bills, invoices, and other vouchers or certified copies
thereof if originals be lost, at such reasonable time and place as
may be designated by the Company or its representative and
shall permit extracts and copies thereof to be made.
No claim under this policy shall be payable unless the terms of
this condition have been complied with.
Usiphil, after the occurrence of the fire, immediately notified petitioner thereof.
Thereafter, private respondent submitted the following documents: (1) Sworn
Statement of Loss and Formal Claim and; (2) Proof of Loss. The submission of
these documents, to the Court's mind, constitutes substantial compliance with
the above provision. As regards the submission of documents to prove
loss, substantial, not strict, compliance with the requirements will
always be deemed sufficient.
In any case, Finman (formerly Summa Insurance) itself acknowledged its liability
when through its Finance Manager, Rosauro Maghirang, it signed the document
indicating that the amount due Usiphil is P842,683.40. Even assuming that
plaintiff-appellee indeed failed to submit certain required documents as proof of
loss per Section 13, such violation was waived by the insurer Summa when it
signed the document-breakdown of the amount due to plaintiff-appellee on the
insurance claim. By such act, defendant-appellant acknowledged its liability under
the insurance policy. Finman alleges that Maghirang was without authority to sign
and therefore without authority to bind Finman. Untenable. At a meeting between
Usiphil's corporate president Pedro Pallalos and Finmans Joaquin Ortega, the
latter summoned Rosauro Maghirang to reconcile the claims of plaintiff-appellee.

46
One who clothes another with apparent authority as his agent and holds him to
the public as such, cannot later be allowed to deny the authority of such person
to act as his agent when such third person entered into the contract in good faith
and in an honest belief that he is such agent.
2. Yes. It is authorized by Sections 243 and 244 of the Insurance Code.
SECTION 243. The amount of any loss or damage for which an
insurer may be liable, under any policy other than life insurance
policy, shall be paid within thirty days after proof of loss is
received by the insurer and ascertainment of the loss or damage
is made either by agreement between the insured and the insurer
or by arbitration; but if such ascertainment is not had or made
within sixty days after such receipt by the insurer of the proof of
loss, then the loss or damage shall be paid within ninety days
after such receipt. Refusal or failure to pay the loss or damage
within the time prescribed herein will entitle the assured to collect
interest on the proceeds of the policy for the duration of the delay
at the rate of twice the ceiling prescribed by the Monetary
Board, unless such failure or refusal to pay is based on the
ground that the claim is fraudulent.
SECTION 244. In case of any litigation for the enforcement of any
policy or contract of insurance, it shall be the duty of the
Commissioner or the Court, as the case may be, to make a finding
as to whether the payment of the claim of the insured has been
unreasonably denied or withheld; and in the affirmative case, the
insurance company shall be adjudged to pay damages which shall
consist of attorney's fees and other expenses incurred by the
insured person by reason of such unreasonable denial or
withholding of payment plus interest of twice the ceiling
prescribed by the Monetary Board of the amount of the claim
due the insured, from the date following the time prescribed in
section two hundred forty-two or in section two hundred fortythree, as the case may be, until the claim is fully
satisfied: Provided, That the failure to pay any such claim within
the time prescribed in said sections shall be considered prima
facie evidence of reasonable delay in payment.
Notably, under Section 244, a prima facie evidence of unreasonable delay in
payment of the claim is created by the failure of the insurer to pay the claim
within the time fixed in both Sections 243 and 244. Further, Section 29 of the
policy itself provides for the payment of such interest:

Part 2_Case Digests


3.1_Insurance Law

Settlement of claim clause. The amount of any loss or damage for


which the company may be liable, under this policy shall be
paid within thirty days after proof of loss is received by
the company and ascertainment of the loss or damage is
made either in an agreement between the insured and the
company or by arbitration; but if such ascertainment is not had or
made within sixty days after such receipt by the company of the
proof of loss, then the loss or damage shall be paid within ninety
days after such receipt. Refusal or failure to pay the loss or
damage within the time prescribed herein will entitle the
assured to collect interest on the proceeds of the policy
for the duration of the delay at the rate of twice the
ceiling prescribed by the Monetary Board unless such failure
or refusal to pay is based on the grounds (sic) that the claim is
fraudulent.
The policy itself obliges petitioner to pay the insurance claim within 30 days after
proof of loss and ascertainment of the loss made in an agreement. The amount
due private respondent was P842,683.40 on April 2, 1985. Finman had until May
2, 1985 to pay Usiphils insurance. For its failure to do so, the CA and the trial
court rightfully directed petitioner to pay, inter alia, 24% interest per annum.

47
P50,000.00 as and for third party liability. The IH Scout was insured with Malayan
Insurance Co.
Even before summons could be served, respondent judge issued an order:
The second incident is the prayer for an order of this court for the
Insurance Company, Perla Compania de Seguros, Inc., to pay
immediately the P5,000 under the "no fault clause" as provided
for under Section 378 of the Insurance Code, and finding that the
requisite documents to be attached in the record, the said
Insurance Company is therefore directed to pay the plaintiffs
(private respondents herein) within 5 days from receipt of this
order.
Perla denied in its Answer its alleged liability under the "no fault indemnity"
provision. It held that under Sec. 378 of the Insurance Code, the insurer liable to
pay the P5,000 is the insurer of the vehicle in which private respondents were
riding, not petitioner, as the provision states that "in the case of an occupant of a
vehicle, claim shall lie against the insurer of the vehicle in which the occupant is
riding, mounting or dismounting from." Respondent judge, however, denied
reconsideration. The judge ordered the issuance of a writ of execution. Hence, the
instant petition praying principally for the annulment and setting aside of
respondent judge's orders. SC issued TRO.
Issue: WON Perla is the insurer liable to indemnify private respondents under
Sec. 378 of the Insurance Code.

P. Compulsory Motor Vehicle Liability Insurance


GR No. L-49699, 1988
71) PERLA COMPANIA de SEGUROS, INC., petitioner, vs. HON. CONSTANTE
A. ANCHETA, Presiding Judge of the Court of First Instance of Camarines
Norte, Branch III, ERNESTO A. RAMOS and GOYENA ZENAROSA-RAMOS,
for themselves and as Guardian Ad Litem for Minors JOBET, BANJO,
DAVID and GRACE all surnamed RAMOS, FERNANDO M. ABCEDE, SR., for
himself and Guardian Ad Litem for minor FERNANDO G. ABCEDE, JR.,
MIGUEL JEREZ MAGO as Guardian Ad Litem for minors ARLEEN R. MAGO,
and ANACLETA J. ZENAROSA, respondents
Facts: On December 27, 1977, the IH Scout (in which private respondents were
riding) collided with a Superlines bus along the national highway in Sta. Elena,
Camarines Norte. Private respondents sustained physical injuries in varying
degrees of gravity. Thus, they filed with the CFI a complaint for damages against
Superlines, the bus driver and petitioner, the insurer of the bus. The bus was
insured with Perla Compania for P50,000 as and for passenger liability and

Held: No. Irrespective of whether or not fault or negligence has with the driver of
the Superlines bus, as private respondents were not occupants of the bus, they
cannot claim the "no fault indemnity" provided in Sec. 378 from Perla. The claim
should be made against the insurer of the vehicle they were riding (i.e., Malayan).
This is very clear from the law. In ordering Perla to pay private respondents the
"no fault indemnity," respondent judge gravely abused his discretion in a manner
that amounts to lack of jurisdiction. The issuance of the corrective writ of
certiorari is therefore warranted.
The key to the resolution of the issue is Sec. 378, which provides:
Any claim for death or injury to any passenger or third party
pursuant to the provisions of this chapter shall be paid without
the necessity of proving fault or negligence of any kind. Provided,
that for purposes of this section
(i) The indemnity in respect of any one person shall not exceed
five thousand pesos;
(ii) The following proofs of loss, when submitted under oath, shall
be sufficient evidence to substantiate the claim:

Part 2_Case Digests


3.1_Insurance Law

(a)Police report of accident, and


(b)Death certificate and evidence sufficient to establish the
proper payee, or
(c)Medical report and evidence of medical or hospital
disbursement in respect of which refund is claimed;
(iii) Claim may be made against one motor vehicle only. In the
case of an occupant of a vehicle, claim shall lie against
the insurer of the vehicle in which the occupant is riding,
mounting or dismounting from. In any other case, claim
shall lie against the insurer of the directly offending
vehicle. In all cases, the right of the party paying the claim to
recover against the owner of the vehicle responsible for the
accident shall be maintained.
From a reading of the provision, the following rules on claims under the "no fault
indemnity" provision, where proof of fault or negligence is not necessary for
payment of any claim for death or injury to a passenger or a third party, are
established:
1. A claim may be made against one motor vehicle only.
2. If the victim is an occupant of a vehicle, the claim shall lie against the
insurer of the vehicle in which he is riding, mounting or dismounting from.
3. In any other case (i.e. if the victim is not an occupant of a vehicle), the
claim shall lie against the insurer of the directly offending vehicle.
4. In all cases, the right of the party paying the claim to recover against the
owner of the vehicle responsible for the accident shall be maintained.
The law is very clear the claim shall lie against the insurer of the vehicle in
which the "occupant" is riding, and no other. The claimant is not free to
choose from which insurer he will claim the "no fault indemnity," as the
law, by using the word "shall", makes it mandatory that the claim be made
against the insurer of the vehicle in which the occupant is riding, mounting or
dismounting from.
That said vehicle might not be the one that caused the accident is of no moment
since the law itself provides that the party paying the claim under Sec. 378 may
recover against the owner of the vehicle responsible for the accident. This is
precisely the essence of "no fault indemnity" insurance which was introduced
to and made part of our laws in order to provide victims of vehicular
accidents or their heirs immediate compensation, although in a limited
amount, pending final determination of who is responsible for the
accident and liable for the victims' injuries or death. In turn, the "no
fault indemnity" provision is part and parcel of the Insurance

48
Code provisions on compulsory motor vehicle liability insurance [Sec.
373-389] and should be read together with the requirement for compulsory
passenger and/or third party liability insurance [Sec. 377] which was mandated in
order to ensure ready compensation for victims of vehicular accidents.
72) FIRST QUEZON CITY INSURANCE COMPANY, INC. vs THE HON. COURT
OF APPEALS and DE DIOS MARIKINA TRANSPORTATION CO. LLORAD

FACTS: On June 10, 1984, Plaintiff Jose V. del Rosario proceeded to the loading
and unloading zone for public utility bus stop. As it approach the bus stop, the bus
slowed down with all its doors wide open: while moving at a crawling pace, i.e., as
slow as an "ordinary walk," it was taking several passengers, about five or seven
of them including the plaintiff, all of whom managed to board the bus while it was
already at the bus stop; plaintiff was the last one to board the bus.
While the plaintiff was still on the bus' running board with his hand on the bus
door's handle bar, the slowly moving bus sped forward at a high speed, as a
result of which, the plaintiff lost his balance and fell from the bus. As plaintiff
clung instinctively to the handle bar, he was dragged by the bus along the
asphalted road for about two (2) seconds. Thereafter, the plaintiff was brought to
the Manila Sanitarium and Hospital where he was given immediate medical
treatment at the emergency ward. The doctors performed a major surgical
operation on plaintiff's right leg.
Plaintiff was confined at the hospital for a total period of forty (40). During his
stay at the hospital, plaintiff incurred medical expenses in the total amount of
P69,444.41. Also, the plaintiff incurred lost earning by way of unearned salaries
amounting to P7,500 due to said physical injuries and the consequent hospital
confinement..
ISSUE: Up to what extent is the insurers liability?
RULING: The insurance company clearly passed the maximum limit of the
petitioner's liability for damages arising from death or bodily injury at P12,000 per
passenger and its maximum liability per accident at P50,000.00. Since only one
passenger was injured in the accident, the insurer's liability for the damages

Part 2_Case Digests


3.1_Insurance Law

suffered by said passenger is pegged to the amount of P12,000 only. What does
the limit of P50,000 per accident mean? It means that the insurer's
liability for any single accident will not exceed P50,000 regardless of the
number of passengers killed or injured therein. For example, if ten (10)
passengers had been injured by the operation of the insured bus, the insurer's
liability for the accident would not be P120,000 (at the rate of P12,000 per
passenger) but would be limited to only P50,000.00 for the entire accident, as
provided in the insurance contract.
The bus company may not recover from the insurance company (herein
petitioner) more than P 12,000.00 per passenger killed or injured, or fifty
thousand (P50,000.00) pesos per accident even if under the judgment of the
court, the erring bus operator will have to pay more than P12,000.00 to each
injured passenger. The trial court's interpretation of the insurance contract was
the correct interpretation.
73) PERLA COMPANIA DE SEGUROS, INC., petitioner, vs. HON. JOSE R.
RAMOLETE, PRIMITIVA Y. PALMES, HONORATO BORBON, SR., OFFICE OF
THE PROVINCIAL SHERIFF, PROVINCE OF CEBU, respondents
Facts: On June 1976, a Cimarron PUJ owned by Nelia Enriquez, and driven by
Cosme Casas, was travelling from Cebu City to Danao City. While passing through
Liloan, Cebu, the Cimarron PUJ collided with a private jeep owned by the late
Calixto Palmes (husband of private respondent Primitiva Palmes) who was then
driving the private jeep. The impact of the collision was such that the private jeep
was flung away to a distance of about thirty (30) feet and then fell on its right
side pinning down Calixto Palmes. He died as a result of cardio-respiratory arrest
due to a crushed chest. The accident also caused physical injuries on the part of
2-year-old Adeudatus Borbon.
Private respondents Primitiva and Honorato Borbon, Sr. (father of Adeudatus) filed
a complaint against Cosme and Nelia before the then Cebu CFI claiming actual,
moral, nominal and exemplary damages as a result of the accident. The claim of
Borbon, Sr. was excluded from the complaint due to jurisdiction.
The CFI ruled in favor of Primitiva, ordering common carrier Nelia to pay her
damages and attorneys fees. The judgment of the trial court became final and
executory and a writ of execution was issued, which however, returned
unsatisfied, prompting the court to summon and examine Nelia. She declared that
the Cimarron PUJ was covered by a third-party liability insurance policy issued by
petitioner Perla.

49
Palmes then filed a motion for garnishment praying that an order of garnishment
be issued against the insurance policy issued by petitioner in favor of the
judgment debtor. Respondent Judge then issued an Order directing the Provincial
Sheriff or his deputy to garnish the third-party liability insurance policy. Petitioner
filed for MR and quashal of the writ of garnishment on the ground that Perla was
not a party to the case and that jurisdiction over its person had never been
acquired by the trial court by service of summons or by any process. The trial
court denied petitioners motion.An Order for issuance of an alias writ of
garnishment was subsequently issued.
More than two (2) years later, the present Petition for Certiorari and Prohibition
was filed with this Court alleging grave abuse of discretion on the part of
respondent Judge Ramolete in ordering garnishment of the third-party liability
insurance contract issued by petitioner Perla in favor of the judgment debtor,
Nelia Enriquez. The Petition should have been dismissed forthwith for having been
filed way out of time but, for reasons which do not appear on the record, was
nonetheless entertained.
Issues:
1. W/N there is GADALEJ on the part of the respondent judge
2. W/N there insurance policy may be subject to garnishment
Held:
1. No. The SC found no grave abuse of discretion or act in excess of or without
jurisdiction on the part of respondent Judge Ramolete in ordering the garnishment
of the judgment debtors third-party liability insurance.
2. Yes. Garnishment has been defined as a species of attachment for reaching
any property or credits pertaining or payable to a judgment debtor. In legal
contemplation, it is a forced novation by the substitution of creditors: the
judgment debtor, who is the original creditor of the garnishee is, through service
of the writ of garnishment, substituted by the judgment creditor who thereby
becomes creditor of the garnishee. Garnishment has also been described as a
warning to a person having in his possession property or credits of the judgment
debtor, not to pay the money or deliver the property to the latter, but rather to
appear and answer the plaintiffs suit.
In order that the trial court may validly acquire jurisdiction to bind the
person of the garnishee, it is not necessary that summons be served
upon him. The garnishee need not be impleaded as a party to the case.
All that is necessary for the trial court lawfully to bind the person of the
garnishee or any person who has in his possession credits belonging to
the judgment debtor is service upon him of the writ of garnishment.

Part 2_Case Digests


3.1_Insurance Law

Rule 39, Section 15 and Rule 57, Section 7(e) of the ROC themselves do not
require that the garnishee be served with summons or impleaded in the case in
order to make him liable.
In the present case, there can be no doubt, therefore, that the trial court actually
acquired jurisdiction over petitioner Perla when it was served with the writ of
garnishment of the third-party liability insurance policy it had issued in favor of
judgment debtor Nelia Enriquez. Perla cannot successfully evade liability thereon
by such a contention.
In a third-party liability insurance contract, the insurer assumes the obligation of
paying the injured third party to whom the insured is liable. The insurer becomes
liable as soon as the liability of the insured to the injured third person attaches.
Prior payment by the insured to the injured third person is not necessary in order
that the obligation of the insurer may arise. From the moment that the insured
became liable to the third person, the insured acquired an interest in the
insurance contract, which interest may be garnished like any other credit.
A separate action is not necessary to establish petitioners liability.
Petition for Certiorari and Prohibition is hereby DISMISSED for having been filed
out of time and for lack of merit. Judgment AFFIRMED
74) GSIS VS CA 308 SCRA 59, June 21, 1999
FACTS: National Food Authority (NFA, formerly National Grains Authority) was the
owner of a Chevrolet truck which was insured against liabilities for death of and
injuries to third persons with the GSIS. Said truck driven by Guillermo Corbeta
collided with a public utility vehicle, a Toyota Tamaraw. The Toyota Tamaraw was
owned and operated by Victor Uy, under the name and style of Victory Line. As a
result, Five (5) passengers died[4] while ten (10) others sustained bodily
injuries. Among those injured were private respondents, Victoria Jaime Vda. de
Kho and Gloria Kho Vda. de Calabia. Among the dead were Maxima Ugmad Vda.
de Kho, Roland Kho and Willie Calabia, Sr.

50
(3)Instituted by herein private respondents against the following: NFA and
Corbeta for damages due to quasi-delict; GSIS as insurer of the truck; Uy for
breach of contract of carriage; and MIGC as insurer of the Toyota Tamaraw.
These cases were later on transferred to Branch II of the Regional Trial Court of
Butuan City. The court rendered its decision [5] holding that Corbetas negligence
was the proximate cause of the collision. The findings of the trial court stated that
the truck which crossed over to the other lane was speeding because after the
collision, its left front wheel was detached and the truck traveled for about fifty
(50) meters and fell into a ravine.
ISSUE: Whether the respondent court erred in holding GSIS solidarily liable with
the negligent insured/owner-operator of the Chevrolet truck for damages awarded
to private respondents which are beyond the limitations of the insurance policy
and the Insurance Memorandum Circular No. 5-78.
RULING: Petitioners position insofar as joint liability is concerned is not tenable. It
is now established that the injured or the heirs of a deceased victim of a vehicular
accident may sue directly the insurer of the vehicle. Note that common carriers
are required to secure Compulsory Motor Vehicle Liability Insurance [CMVLI]
coverage as provided under Sec. 374 [13] of the Insurance Code, precisely for the
benefit of victims of vehicular accidents and to extend them immediate relief
Compulsory Motor Vehicle Liability Insurance (third party liability, or TPL) is
primarily intended to provide compensation for the death or bodily injuries
suffered by innocent third parties or passengers as a result of a negligent
operation and use of motor vehicles. The victims and/or their defendants
[dependents] are assured of immediate financial assistance, regardless of the
financial capacity of motor vehicle owners.
xxx

Three (3) cases were filed with the Court of First Instance:
(1) commenced by Uy against NFA and Corbeta.
(2) For damages, was filed by an injured passenger, Librado Taer, against Uy, the
operator of the public utility vehicle, and insurer, Mabuhay Insurance and
Guaranty Co. (MIGC).

The injured for whom the contract of insurance is intended can sue directly the
insurer. The general purpose of statutes enabling an injured person to proceed
directly against the insurer is to protect injured persons against the insolvency of
the insured who causes such injury, and to give such injured person a certain
beneficial interest in the proceeds of the policy, and statutes are to be liberally
construed so that their intended purpose may be accomplished. It has even been

Part 2_Case Digests


3.1_Insurance Law

51

held that such a provision creates a contractual relation which inures to the
benefit of any and every person who may be negligently injured by the named
insured as if such injured person were specifically named in the policy.
However, although the victim may proceed directly against the insurer for
indemnity, the third party liability is only up to the extent of the insurance policy
and those required by law. While it is true that where the insurance contract
provides for indemnity against liability to third persons, and such third persons
can directly[17] sue the insurer, the direct liability of the insurer under indemnity
contracts against third party liability does not mean that the insurer can be held
liable in solidum with the insured and/or the other parties found at fault. [18] For the
liability of the insurer is based on contract; that of the insured carrier or vehicle
owner is based on tort.[19] The liability of GSIS based on the insurance contract is
direct, but not solidary with that of the NFA. The latters liability is based
separately on Article 2180[20] of the Civil Code.[21]
Obviously, the insurer could be held liable only up to the extent of what was
provided for by the contract of insurance, in accordance with CMVLI law. At the
time of the incident, the schedule of indemnities for death and/or bodily injuries,
professional fees, hospital and other charges payable under a CMVLI coverage
was provided under the Insurance Memorandum Circular (IMC) No. 5-78 which
was approved on November 10, 1978, at twelve thousand (P12,000.00) pesos
per victim.
Consequently, heirs of the victims who died in the May 9, 1979 vehicular
incident, could proceed (1) against GSIS for the indemnity of P12,000 for each
dead victim, and against NFA and Guillermo Corbeta for any other damages or
expenses claimed; or (2) against NFA and Corbeta to pay them all their claims in
full.
It follows also that injured victims, Gloria Kho Vda. de Calabia and Victoria
Kho, could claim their medical expenses for eight thousand nine hundred thirtyfive pesos and six centavos (P8,935.06) and eight hundred thirty-two (P832.00)
pesos, from any of the following: GSIS, NFA, or Corbeta. As to the other damages,
only NFA or Corbeta may be held liable therefor.

Q. Jurisdiction of Insurance Commission


75) PHILIPPINE AMERICAN LIFE INSURANCE COMPANY and RODRIGO DE
LOS
REYESvs.
HON. ARMANDO ANSALDO, in his capacity as Insurance Commissioner,
and RAMON MONTILLA PATERNO, JR
FACTS: April 17, 1986- a letter-complaint of private respondent Ramon M.
Paterno, Jr. to respondent Commissioner, alleging certain problems encountered
by agents, supervisors, managers and public consumers of the Philippine
American Life Insurance Company (Philamlife) as a result of certain practices by
said company.
April 29, 1986- to respondent Commissioner, petitioner De los Reyes suggested
that private respondent "submit some sort of a 'bill of particulars' listing and
citing actual cases, facts, dates, figures, provisions of law, rules and regulations,
and all other pertinent data which are necessary to enable him to prepare an
intelligent reply" .A copy of this letter was sent by the Insurance Commissioner to
private respondent for his comments thereon.

Part 2_Case Digests


3.1_Insurance Law

May 16, 1986- respondent Commissioner received a letter from private


respondent maintaining that his letter-complaint of April 17, 1986 was sufficient in
form and substance, and requested that a hearing thereon be conducted.
June 6, 1986- Petitioner De los Reyes, in his letter to respondent Commissioner
dated, reiterated his claim that private respondent's letter of May 16, 1986 did
not supply the information he needed to enable him to answer the lettercomplaint.

52
RULING: The general regulatory authority of the Insurance Commissioner is
described in Section 414 of the Insurance Code, to wit:
The Insurance Commissioner shall have the duty to see that all laws relating to
insurance, insurance companies and other insurance matters, mutual benefit
associations and trusts for charitable uses are faithfully executed and to perform
the duties imposed upon him by this Code, . . .
On the other hand, Section 415 provides:

July 14- a hearing on the letter-complaint was held by respondent Commissioner


on the validity of the Contract of Agency complained of by private respondent.
August 4- private respondent submitted a letter of specification to respondent
Commissioner dated July 31, 1986, reiterating his letter of April 17, 1986 and
praying that the provisions on charges and fees stated in the Contract of Agency
executed between Philamlife and its agents, as well as the implementing
provisions as published in the agents' handbook, agency bulletins and circulars,
be declared as null and void. He also asked that the amounts of such charges and
fees already deducted and collected by Philamlife in connection therewith be
reimbursed to the agents, with interest at the prevailing rate reckoned from the
date when they were deducted.
October 14, 1986- Manuel Ortega, Philamlife's Senior Assistant Vice-President and
Executive Assistant to the President, asked that respondent Commission first rule
on the questions of the jurisdiction of the Insurance Commissioner over the
subject matter of the letters-complaint and the legal standing of private
respondent.

In addition to the administrative sanctions provided elsewhere in this Code, the


Insurance Commissioner is hereby authorized, at his discretion, to impose upon
insurance companies, their directors and/or officers and/or agents, for any willful
failure or refusal to comply with, or violation of any provision of this Code, or any
order, instruction, regulation or ruling of the Insurance Commissioner, or any
commission of irregularities, and/or conducting business in an unsafe and
unsound manner as may be determined by the the Insurance Commissioner, the
following:
(a) fines not in excess of five hundred pesos a day; and
(b) suspension, or after due hearing, removal of directors and/or officers and/or
agents.
A plain reading of the above-quoted provisions show that the Insurance
Commissioner has the authority to regulate the business of insurance,
which is defined as follows:

ISSUE: Whether or not the resolution of the legality of the Contract of Agency
falls within the jurisdiction of the Insurance Commissioner.

(2) The term "doing an insurance business" or "transacting an insurance


business," within the meaning of this Code, shall include

Private respondent contends that the Insurance Commissioner has jurisdiction to


take cognizance of the complaint in the exercise of its quasi-judicial powers. The
Solicitor General, upholding the jurisdiction of the Insurance Commissioner,
claims that under Sections 414 and 415 of the Insurance Code, the Commissioner
has authority to nullify the alleged illegal provisions of the Contract of Agency.

(a) making or proposing to make, as insurer, any insurance contract;


(b) making, or proposing to make, as surety, any contract of suretyship as a
vocation and not as merely incidental to any other legitimate business or
activity of the surety;

Part 2_Case Digests


3.1_Insurance Law

(c) doing any kind of business, including a reinsurance business, specifically


recognized as constituting the doing of an insurance business within the
meaning of this Code;
(d) doing or proposing to do any business in substance equivalent to any of the
foregoing in a manner designed to evade the provisions of this Code.
(Insurance Code, Sec. 2[2])
Since the contract of agency entered into between Philamlife and its
agents is not included within the meaning of an insurance business,
Section 2 of the Insurance Code cannot be invoked to give jurisdiction
over the same to the Insurance Commissioner. Expressio unius est exclusio
alterius.
ADDITIONAL PRONOUNCEMENTS: (With regard to private respondent's
contention that the quasi-judicial power of the Insurance Commissioner under
Section 416 of the Insurance Code applies in his case), we likewise rule in the
negative.
Section 416 of the Code, provides:
The Commissioner shall have the power to adjudicate claims and complaints
involving any loss, damage or liability for which an insurer may be answerable

53
under any kind of policy or contract of insurance, or for which such insurer may
be liable under a contract of suretyship, or for which a reinsurer may be used
under any contract or reinsurance it may have entered into, or for which a mutual
benefit association may be held liable under the membership certificates it has
issued to its members, where the amount of any such loss, damage or liability,
excluding interest, costs and attorney's fees, being claimed or sued upon any kind
of insurance, bond, reinsurance contract, or membership certificate does not
exceed in any single claim one hundred thousand pesos.
A reading of the said section shows that the quasi-judicial power of the
Insurance Commissioner is limited by law "to claims and complaints
involving any loss, damage or liability for which an insurer may be
answerable under any kind of policy or contract of insurance, . . ."
Hence, this power does not cover the relationship affecting the
insurance company and its agents but is limited to adjudicating claims
and complaints filed by the insured against the insurance company.

S-ar putea să vă placă și